[obm-l] Re: [obm-l] Comunicação

2023-08-26 Por tôpico Esdras Muniz
Seria muito legal se existisse.

Em sex, 25 de ago de 2023 18:24, Priscila Santana 
escreveu:

>
> Olá!
>
> Existe algum grupo de discussão de questões olímpicas no WhatsApp?
>
> Atte.
>
> *Priscila S. da Paz*
>
>
>
> --
> Esta mensagem foi verificada pelo sistema de antivírus e
> acredita-se estar livre de perigo.
>

-- 
Esta mensagem foi verificada pelo sistema de antiv�rus e
 acredita-se estar livre de perigo.



[obm-l] Re: [obm-l] Re: [obm-l] Uma recorrência diferente

2023-04-06 Por tôpico Esdras Muniz
Vc pode pegar a função geratriz e usar a fórmula de Ramanujan pra calcular
o termo geral. Acredito que a função geratriz seja: e^{x^2/2+x}, já na
forma (x_n)x^n/n!...

Em qui, 6 de abr de 2023 19:03, Carlos Gustavo Tamm de Araujo Moreira <
g...@impa.br> escreveu:

> Caro Vanderlei,
> Não parece haver uma fórmula fechada muito simples. Veja
> https://oeis.org/A85 para várias referências
> sobre essa sequência.
> Abraços,
> Gugu
>
> On Wed, Apr 5, 2023 at 11:41 PM Professor Vanderlei Nemitz <
> vanderma...@gmail.com> wrote:
>
>> Oi, mestres!
>>
>> Estava resolvendo um problema de combinatória e obtive essa recorrência:
>>
>> *x(n) = x(n - 1) + (n - 1).x(n - 2), com x1 = 1 e x2 = 2*.
>>
>> Por exemplo, x3 = x2 + 2.x1 e x9 = x8 + 8.x7
>>
>> Como resolver quando os coeficientes não são todos constantes?
>>
>> Apenas como curiosidade, o problema que originou a recorrência é:
>> (IME - RJ) - Um professor dá um teste surpresa para uma turma de 9
>> alunos, e diz que o teste pode ser feito sozinho ou em grupos de 2 alunos.
>> De quantas formas a turma pode ser organizar para fazer o teste? (Por
>> exemplo, uma turma de 3 alunos pode ser organizar de 4 formas e uma turma
>> de 4 alunos pode se organizar de 10 formas)
>>
>>
>>
>> 
>>  Não
>> contém vírus.www.avast.com
>> 
>> <#m_-6699161896707809793_m_4118911927943204904_DAB4FAD8-2DD7-40BB-A1B8-4E2AA1F9FDF2>
>>
>> --
>> Esta mensagem foi verificada pelo sistema de antivírus e
>> acredita-se estar livre de perigo.
>
>
> --
> Esta mensagem foi verificada pelo sistema de antivírus e
> acredita-se estar livre de perigo.

-- 
Esta mensagem foi verificada pelo sistema de antiv�rus e
 acredita-se estar livre de perigo.



[obm-l] Re: [obm-l] Seria por distribuição binomial ou alguma recorrência

2023-02-28 Por tôpico Esdras Muniz
Letra E na verdade 

Em ter., 28 de fev. de 2023 às 11:52, Bianca Flores 
escreveu:

> Alguém poderia ajudar com essa questão: estou frustrada porque não consigo
> chegar ao gabarito E.
>
> Um estudante preenche, aleatoriamente e de forma independente cada uma das
> questões, um exame de múltipla escolha com 5 respostas possíveis (das quais
> apenas uma é correta) para cada uma de 25 questões. A probabilidade que ele
> acerte um número par de questões é dada por:
>
> (A)(1-(4/5)^25)/2
> (B)(1-(3/5)^25)/2
> (C)((3/5)^25)/2
> (D)(1+(4/5)^25)/2
> (E)(1+(3/5)^25)/2
>
> Tento de todas as formar usar a distribuição binomial, alguma recorrência,
> mas sem sucesso.
> Bianca
>
> --
> Esta mensagem foi verificada pelo sistema de antivírus e
> acredita-se estar livre de perigo.
>
-- 
Esdras Muniz Mota
Mestrando em Matemática
Universidade Federal do Ceará

-- 
Esta mensagem foi verificada pelo sistema de antiv�rus e
 acredita-se estar livre de perigo.



[obm-l] Re: [obm-l] Seria por distribuição binomial ou alguma recorrência

2023-02-28 Por tôpico Esdras Muniz
Veja 1 como a soma de todas as probabilidades possíveis e (3/5)^25 como a
probabilidade de ele acertas uma quantidade par menos a probabilidade de
ele acertar uma quantidade ímpar.

Em ter., 28 de fev. de 2023 às 11:58, Esdras Muniz <
esdrasmunizm...@gmail.com> escreveu:

> Acredito que é letra B. Vc pode fazer usando binômio de Newton…
>
> Em ter., 28 de fev. de 2023 às 11:52, Bianca Flores 
> escreveu:
>
>> Alguém poderia ajudar com essa questão: estou frustrada porque não
>> consigo chegar ao gabarito E.
>>
>> Um estudante preenche, aleatoriamente e de forma independente cada uma
>> das questões, um exame de múltipla escolha com 5 respostas possíveis (das
>> quais apenas uma é correta) para cada uma de 25 questões. A probabilidade
>> que ele acerte um número par de questões é dada por:
>>
>> (A)(1-(4/5)^25)/2
>> (B)(1-(3/5)^25)/2
>> (C)((3/5)^25)/2
>> (D)(1+(4/5)^25)/2
>> (E)(1+(3/5)^25)/2
>>
>> Tento de todas as formar usar a distribuição binomial, alguma
>> recorrência, mas sem sucesso.
>> Bianca
>>
>> --
>> Esta mensagem foi verificada pelo sistema de antivírus e
>> acredita-se estar livre de perigo.
>>
> --
> Esdras Muniz Mota
> Mestrando em Matemática
> Universidade Federal do Ceará
>
>
> --
Esdras Muniz Mota
Mestrando em Matemática
Universidade Federal do Ceará

-- 
Esta mensagem foi verificada pelo sistema de antiv�rus e
 acredita-se estar livre de perigo.



[obm-l] Re: [obm-l] Seria por distribuição binomial ou alguma recorrência

2023-02-28 Por tôpico Esdras Muniz
Acredito que é letra B. Vc pode fazer usando binômio de Newton…

Em ter., 28 de fev. de 2023 às 11:52, Bianca Flores 
escreveu:

> Alguém poderia ajudar com essa questão: estou frustrada porque não consigo
> chegar ao gabarito E.
>
> Um estudante preenche, aleatoriamente e de forma independente cada uma das
> questões, um exame de múltipla escolha com 5 respostas possíveis (das quais
> apenas uma é correta) para cada uma de 25 questões. A probabilidade que ele
> acerte um número par de questões é dada por:
>
> (A)(1-(4/5)^25)/2
> (B)(1-(3/5)^25)/2
> (C)((3/5)^25)/2
> (D)(1+(4/5)^25)/2
> (E)(1+(3/5)^25)/2
>
> Tento de todas as formar usar a distribuição binomial, alguma recorrência,
> mas sem sucesso.
> Bianca
>
> --
> Esta mensagem foi verificada pelo sistema de antivírus e
> acredita-se estar livre de perigo.
>
-- 
Esdras Muniz Mota
Mestrando em Matemática
Universidade Federal do Ceará

-- 
Esta mensagem foi verificada pelo sistema de antiv�rus e
 acredita-se estar livre de perigo.



[obm-l] Re: [obm-l] Séries

2022-08-25 Por tôpico Esdras Muniz
Me manda.

Em qui, 25 de ago de 2022 17:36, Israel Meireles Chrisostomo <
israelmchrisost...@gmail.com> escreveu:

> Olá pessoal, recentemente eu tive umas ideias sobre séries envolvendo o
> número e (napier), o seno e o cosseno.Alguém por favor poderia me
> corrigir?São ideias originais e séries infinitas nunca antes pensadas.
> Alguém por favor me ajuda a corrigir.Ver se estou viajandoMeu desejo é
> que vcs digam que esteja certo, sejam pacientes por favor.Quem se dispor,
> por favor, chama inbox
>
> -
> Somente a Deus Glória.
> Israel Meireles Chrisostomo
>
> --
> Esta mensagem foi verificada pelo sistema de antivírus e
> acredita-se estar livre de perigo.

-- 
Esta mensagem foi verificada pelo sistema de antiv�rus e
 acredita-se estar livre de perigo.



[obm-l] Re: [obm-l] Função phi de Euler

2022-07-14 Por tôpico Esdras Muniz
Quis dizer φ(p)=p-1.

Em qui, 14 de jul de 2022 12:02, Esdras Muniz 
escreveu:

> Oi(o)=p-1, aí isso só vale se o primo for da firma 6k+1.
>
> Em qui, 14 de jul de 2022 11:52, Rubens Vilhena Fonseca <
> rubens.vilhen...@gmail.com> escreveu:
>
>> Saudações a todos da lista.
>> É um fato que para primos p ímpares, a função de Euler phi(p)=p-1 é
>> sempre um valor par.
>> Os primos 7, 13, 19, 31, 37, 67, 73, 79, 97, ... tem valores pares
>> múltiplos de 3.
>> Existe algum caminho a tomar para determinar quando phi(p) = 3 .(2k)?
>> Agradeço qualquer solução ou  informação ou indicação de leituras sobre
>> o  problema.
>> Att
>>
>>
>> --
>> Esta mensagem foi verificada pelo sistema de antivírus e
>> acredita-se estar livre de perigo.
>
>

-- 
Esta mensagem foi verificada pelo sistema de antiv�rus e
 acredita-se estar livre de perigo.



[obm-l] Re: [obm-l] Função phi de Euler

2022-07-14 Por tôpico Esdras Muniz
Oi(o)=p-1, aí isso só vale se o primo for da firma 6k+1.

Em qui, 14 de jul de 2022 11:52, Rubens Vilhena Fonseca <
rubens.vilhen...@gmail.com> escreveu:

> Saudações a todos da lista.
> É um fato que para primos p ímpares, a função de Euler phi(p)=p-1 é sempre
> um valor par.
> Os primos 7, 13, 19, 31, 37, 67, 73, 79, 97, ... tem valores pares
> múltiplos de 3.
> Existe algum caminho a tomar para determinar quando phi(p) = 3 .(2k)?
> Agradeço qualquer solução ou  informação ou indicação de leituras sobre o
> problema.
> Att
>
>
> --
> Esta mensagem foi verificada pelo sistema de antivírus e
> acredita-se estar livre de perigo.

-- 
Esta mensagem foi verificada pelo sistema de antiv�rus e
 acredita-se estar livre de perigo.



Re: [obm-l] Re: Polinomio

2022-01-29 Por tôpico Esdras Muniz
O único polinômio limitado é o constante.

Em sáb, 29 de jan de 2022 14:03, Carlos Juarez <
carlosjuarezmart...@gmail.com> escreveu:

> k=p(c)+1 não vale sempre?
>
> Em sáb, 29 de jan de 2022 09:27, Israel Meireles Chrisostomo <
> israelmchrisost...@gmail.com> escreveu:
>
>> Desculpe me o que eu quis dizer é que dado um c real existe um k positivo
>> tal que p(c)>
>> Em sáb., 29 de jan. de 2022 09:12, Israel Meireles Chrisostomo <
>> israelmchrisost...@gmail.com> escreveu:
>>
>>> Olá pessoal.Eu gostaria de saber se um polinomio é limitado, isto é,
>>> dado P(x) existe um k positivo tal que P(x)>>
>>
>> --
>> Esta mensagem foi verificada pelo sistema de antivírus e
>> acredita-se estar livre de perigo.
>
>
> --
> Esta mensagem foi verificada pelo sistema de antivírus e
> acredita-se estar livre de perigo.

-- 
Esta mensagem foi verificada pelo sistema de antiv�rus e
 acredita-se estar livre de perigo.



[obm-l] Re: [obm-l] Quebra do RSA por solução do problema de fatoração - Eric Campos Bastos Guedes

2022-01-11 Por tôpico Esdras Muniz
Bem, eu não sou especialista no assunto, mas uma observação óbvia é que
para tentar na força bruta fatorar N, vc vai usar no máximo 2√N/ln(N)
divisões (pelo teorema dos números primos). Uma coisa bastante interessante
seria vc mostrar que seu algoritmo faz menos interação que isso, ou ainda
que na maioria dos casos ele é melhor.

Em ter, 11 de jan de 2022 10:03, Eric Campos Bastos Guedes <
ebastosgue...@gmail.com> escreveu:

> Proponho um algoritmo para quebrar o RSA. O algoritmo que eu propus antes
> trabalhava com números muito grandes e por isso podia não funcionar
> direito. Esse trabalha com números bem menores porque usa módulo N numa
> etapa. O algoritmo e sua explicação estão no YouTube com o mesmo título
> desse e-mail. São dois vídeos, o que conta é o mais recente deste ano de
> 2022.
>
> QUEBRA DO RSA - ALGORITMO N.2
>
> PASSO 1: a=3
>
> inicializando o valor de a
>
> PASSO 2: N é o inteiro a ser fatorado
>
> N é o número usado no RSA. N é o produto de dois números primos grandes
> não muito próximos.
>
> PASSO 3: M=N^512 (N elevado a 512)
>
> M é um número grande mas não muito grande. O valor de P não vai
> ultrapassar muito o valor de M. P é uma variável inteira que acumula
> fatores primos. Aí você faz MDC(P, N) para tentar fatorar N.
>
> PASSO 4: a=a+1
>
> O valor de a é atualizado para a+1, isto é,  é  acrescentado 1 ao valor de
> a
>
> PASSO 5: P=a
>
> O valor de P é inicializado
>
> PASSO 6: b = número aleatório entre 0 e 1
> PASSO 7: Se b > 1/2 faça c=1 senão faça c=-1
>
> O objetivo dos passos 6 e 7 é atribuir à variável c um valor que pode ser
> 1 ou -1. Isso nem precisa ser feito de modo aleatório, mas acho que vai
> funcionar melhor se for aleatório.
>
> PASSO 8: P=P(P+c)
>
> É uma atribuição de valor. O novo valor de P passa a ser P(P+c). Note que
> P+c é relativamente primo com P. Na prática são acrescentados novos fatores
> primos a P que vai acumular fatores primos.
>
> PASSO 9: Se P < M vá para o PASSO 6
>
> Esse passo determina um looping para acumular fatores em P.
>
> PASSO 10: Se MDC(P, N) for diferente de 1 vá para o PASSO 14
>
> Se MDC(P, N) for diferente de 1 ele pode ser um fator primo de N. Resta
> verificar se ele não é o próprio N. Isso vai ser feito no PASSO 14.
>
> PASSO 11: P = Resto da divisão de P por N
>
> Esse passo é para trabalharmos com números menores.
>
> PASSO 12: Se P < 4 faça P=4
>
> Talvez esse passo possa ser omitido
>
> PASSO 13: vá para o PASSO 6
>
> PASSO 14: Se MDC(P, N)=N vá para o PASSO 4
>
> Se MDC(P, N) = N não foram encontrados fatores primos e algoritmo recomeça
> do ponto apropriado.
>
> PASSO 15: MDC(P, N) é fator (primo) de N
>
> FIM
>
> Eu fui menção honrosa na Olimpíada Ibero-americana de Matemática
> Universitária em 2006. Acho que este meu trabalho merece ser avaliado.
>
>
>
>
> --
> Esta mensagem foi verificada pelo sistema de antivírus e
> acredita-se estar livre de perigo.

-- 
Esta mensagem foi verificada pelo sistema de antiv�rus e
 acredita-se estar livre de perigo.



[obm-l] Re: [obm-l] cálculo

2021-09-15 Por tôpico Esdras Muniz
O ponto é que tanto o conjunto dos números racionais quanto o conjunto dos
números irracionais são densos em R. Portanto, para todo intervalo não
degenerado, o máximo de f será 1 e o mínimo de f será zero. Daí, a integral
superior será sempre maior que a integral inferior, portanto a função não é
integravel.

Em qua, 15 de set de 2021 00:11, Israel Meireles Chrisostomo <
israelmchrisost...@gmail.com> escreveu:

> Olá pessoal. eu estou me esforçando para entender esse exemplo do
> guidorizzi, alguém poderia me explicar?Aqui vai:
> Seja f uma função, tal que se x é racional então f igual a 1, se x é
> irracional então f igual a zero. Mostre que a função não é riemann
> integrável.
>
> --
> Israel Meireles Chrisostomo
>
> --
> Esta mensagem foi verificada pelo sistema de antivírus e
> acredita-se estar livre de perigo.

-- 
Esta mensagem foi verificada pelo sistema de antiv�rus e
 acredita-se estar livre de perigo.



Re: [obm-l] Transcendentes

2021-04-01 Por tôpico Esdras Muniz
Isso aí é falso, basta vc pegar a série de Taylor do seno por exemplo e
aplicar o π.

Em qui, 1 de abr de 2021 18:50, Israel Meireles Chrisostomo <
israelmchrisost...@gmail.com> escreveu:

> Como provar que se u é um número transcendentes e a_k são números
> algébricos, para tô natural k, então $u^{m_0}a_0 + u^{m_1}a_1 + u^{m_2}a_2
> + ... + u^{m_n}a_n $ não pode ser zero.onde $m_k$ é um inteiro positivo.
>
> --
> Esta mensagem foi verificada pelo sistema de antivírus e
> acredita-se estar livre de perigo.

-- 
Esta mensagem foi verificada pelo sistema de antiv�rus e
 acredita-se estar livre de perigo.



[obm-l] Re: [obm-l] Usamo ( polinômios )

2020-09-11 Por tôpico Esdras Muniz
Tava dando uma olhada, vi que só com as constantes a e b não dá certo, mas
uma solução que funciona é pegar:  f: (x_1, a_1y_1); (x_2,
a_2y_2);...;(x_{n+1},
a_{n+1}y_{n+1}) e g: (b_1x_1, y_1); (b_2x_2, y_2);...;(x_{n+1},
b_{n+1}y_{n+1}), com com a_i+b_i=2 e não nulos e diferentes. Daí você
mostra que esses a_i e b_i existem.

Em sex, 11 de set de 2020 15:30, Esdras Muniz 
escreveu:

> Acho que é assim: Dado o tal polinômio P(x), de grau n, podemos supor spdg
> que P não tem raiz real (mas não é necessário) tome os pontos (x_1, y_1); 
> (x_2,
> y_2);...;(x_{n+1}, y_{n+1}) sobre o gráfico de P, onde y_i !=0. Então
> sejam f e g respectivamente os polinômios de grau no máximo n que passam
> por f: (x_1, ay_1); (x_2, by_2);...;(x_{n+1}, by_{n+1}) e g: (bx_1, y_1); 
> (ax_2,
> y_2);...;(x_{n+1}, ay_{n+1}), com b<0 Daí, f e g têm n raízes reais cada um (e distintas), e (f+g)/2 coincide
> com P em n+1 pontos, então P=(f+g)/2.
>
> Agora é só mostrar que existem a e b pra que f e g sejam monicos. Pra
> isso, vc faz o polinômio interpolador de P com aqueles n pontos e dá uma
> ajustada na constantes, não é nada difícil.
>
> Em qui, 10 de set de 2020 18:59, Jeferson Almir 
> escreveu:
>
>> (USAMO) Prove que qualquer polinômio mônico de grau n, com coeficientes
>> reais, pode ser escrito como média aritmética de dois polinômios mônicos de
>> grau n com n raízes reais cada.
>>
>> O material sugere usar o polinômio interpolador de Lagrange.
>> Alguém teria uma solução pra isso ?? via polinômio de Lagrange.?
>>
>> --
>> Esta mensagem foi verificada pelo sistema de antivírus e
>> acredita-se estar livre de perigo.
>
>

-- 
Esta mensagem foi verificada pelo sistema de antiv�rus e
 acredita-se estar livre de perigo.



[obm-l] Re: [obm-l] Usamo ( polinômios )

2020-09-11 Por tôpico Esdras Muniz
Acho que é assim: Dado o tal polinômio P(x), de grau n, podemos supor spdg
que P não tem raiz real (mas não é necessário) tome os pontos (x_1, y_1); (x_2,
y_2);...;(x_{n+1}, y_{n+1}) sobre o gráfico de P, onde y_i !=0. Então sejam
f e g respectivamente os polinômios de grau no máximo n que passam por f: (x_1,
ay_1); (x_2, by_2);...;(x_{n+1}, by_{n+1}) e g: (bx_1, y_1); (ax_2,
y_2);...;(x_{n+1}, ay_{n+1}), com b<0
escreveu:

> (USAMO) Prove que qualquer polinômio mônico de grau n, com coeficientes
> reais, pode ser escrito como média aritmética de dois polinômios mônicos de
> grau n com n raízes reais cada.
>
> O material sugere usar o polinômio interpolador de Lagrange.
> Alguém teria uma solução pra isso ?? via polinômio de Lagrange.?
>
> --
> Esta mensagem foi verificada pelo sistema de antivírus e
> acredita-se estar livre de perigo.

-- 
Esta mensagem foi verificada pelo sistema de antiv�rus e
 acredita-se estar livre de perigo.



[obm-l] Re: [obm-l] Re: É um número?

2020-08-27 Por tôpico Esdras Muniz
Para um número n natural, podemos definir a^n como a.a.a...a n vezes. Se
a!=0, a^(-n)=(1/a)^n. E a^(1/m) como o real b tal que b^m=a. Como esse b
nem sempre existe, devemos tomar um certo cuidado. Só não vai ter solução
se a<0 e m for par (é fácil mostrar isso usando polinômios). Daí, seguindo
essa linha de raciocínio, podemos definir naturalmente a^(n/m). Agora
podemos tentar generalizar de forma que a função f(x)=a^x seja contínua.
Daí, como o conjunto dos racionais é denso nós reais e, da forma como foi
definida, essa função definida para x racional é monótona, temos a^x
definido também para x irracional. Mas isso custa um preço, não podemos
mais tomar a negativo. Por exemplo, você poderia dizer que (-1)^(1/3) =-1,
pois (-1)(-1)(-1)=-1. Mas assim, se essa f é contínua, o limite quando n
vai pro infinito de (-1)^((n)/(3n+1)) deveria ser -1, mas esse limite nem
mesmo existe.

Em qui, 27 de ago de 2020 20:00, Maikel Andril Marcelino <
maikel.marcel...@ifrn.edu.br> escreveu:

>
> Marcone, qual das duas opções a < 0 ou x pertencente aos irracionais? Ou
> as duas opções juntas?
>
>
> Atenciosamente,
>
> *Maikel Andril Marcelino*
> *(84) 9-9149-8991 (Contato)*
> *(84) 8851-3451 (WhatsApp)*
>
> --
> *De:* owner-ob...@mat.puc-rio.br  em nome de
> marcone augusto araújo borges 
> *Enviado:* quinta-feira, 27 de agosto de 2020 18:14
> *Para:* obm-l@mat.puc-rio.br
> *Assunto:* [obm-l] É um número?
>
> Faz sentido a^x, se a< 0 e x é irracional positivo?
> --
> Esta mensagem foi verificada pelo sistema de antivírus e
> acredita-se estar livre de perigo.
>
> --
> Esta mensagem foi verificada pelo sistema de antivírus e
> acredita-se estar livre de perigo.
>

-- 
Esta mensagem foi verificada pelo sistema de antiv�rus e
 acredita-se estar livre de perigo.



[obm-l] Re: [obm-l] Re: [obm-l] Sequência das médias ponderadas

2020-08-26 Por tôpico Esdras Muniz
Dado e>0, existe n0 tq m>=n0 então a-e
escreveu:

> Acho que isso tá mal formulado.
> Por exemplo,quanto é s_3?
>
> On Tue, Aug 25, 2020 at 3:49 PM Artur Costa Steiner <
> artur.costa.stei...@gmail.com> wrote:
>
>> Isso me foi dado como verdadeiro, mas ainda não cheguei a uma conclusão.
>>
>> Sejam (a_ n) uma sequência de reais positivos e (s_n) a sequência das
>> médias ponderadas de (a_n,) com relação aos pesos positivos (p_n).
>> Suponhamos que lim p_n = p, 0 < p < oo, e que a sequência das médias
>> aritméticas de (a_n) convirja para o real a. Então, s_n --> a.
>>
>> Abraços
>> Artur
>>
>> --
>> Esta mensagem foi verificada pelo sistema de antivírus e
>> acredita-se estar livre de perigo.
>
>
> --
> Esta mensagem foi verificada pelo sistema de antivírus e
> acredita-se estar livre de perigo.

-- 
Esta mensagem foi verificada pelo sistema de antiv�rus e
 acredita-se estar livre de perigo.



[obm-l] Re: [obm-l] Sequência das médias ponderadas

2020-08-25 Por tôpico Esdras Muniz
Basta ter que as soma dos pesos vai pro infinito. Isso é um exercício do
livro de análise real do Elon.

Em ter, 25 de ago de 2020 15:49, Artur Costa Steiner <
artur.costa.stei...@gmail.com> escreveu:

> Isso me foi dado como verdadeiro, mas ainda não cheguei a uma conclusão.
>
> Sejam (a_ n) uma sequência de reais positivos e (s_n) a sequência das
> médias ponderadas de (a_n,) com relação aos pesos positivos (p_n).
> Suponhamos que lim p_n = p, 0 < p < oo, e que a sequência das médias
> aritméticas de (a_n) convirja para o real a. Então, s_n --> a.
>
> Abraços
> Artur
>
> --
> Esta mensagem foi verificada pelo sistema de antivírus e
> acredita-se estar livre de perigo.

-- 
Esta mensagem foi verificada pelo sistema de antiv�rus e
 acredita-se estar livre de perigo.



[obm-l] Re: [obm-l] Resto da divisão de um polinômio

2020-08-22 Por tôpico Esdras Muniz
Vc pode dizer que x^2=-(x+1) e abrir as contas.

Em sáb, 22 de ago de 2020 21:19, Professor Vanderlei Nemitz <
vanderma...@gmail.com> escreveu:

> Oi!
>
> Existe algum fato específico que ajude a determinar o resto da divisão de
> um polinômio de grau elevado por outro, ou depende do caso?
>
> Por exemplo, como encontrar o seguinte resto, sem excessivos cálculos?
> Muito obrigado!
>
> *Determine o resto da divisão do polinômio x^30 - x^28 + 7x^12 por x^2 +
> x + 1?*
>
>
> 
>  Livre
> de vírus. www.avast.com
> .
> <#m_695281535589841859_DAB4FAD8-2DD7-40BB-A1B8-4E2AA1F9FDF2>
>
> --
> Esta mensagem foi verificada pelo sistema de antivírus e
> acredita-se estar livre de perigo.

-- 
Esta mensagem foi verificada pelo sistema de antiv�rus e
 acredita-se estar livre de perigo.



[obm-l] Re: [obm-l] Re: [obm-l] Re: [obm-l] Re: [obm-l] Re: [obm-l] polinômio irredutível

2020-08-17 Por tôpico Esdras Muniz
E se p=3, e p divide N^2+9, então p^2 divide N^2+9.

Então o critério de Eisenstein realmente não é tão abrangente. Será que tem
algum outro critério que cubra casos em que o de Eisenstein não cubra?

Em seg, 17 de ago de 2020 09:46, Claudio Buffara 
escreveu:

> Boa! Se p <> 3 mas p divide 3N e 3N^2, então p divide N ==> p não divide
> N^3 + 9.
>
> On Sun, Aug 16, 2020 at 10:51 PM Esdras Muniz 
> wrote:
>
>> Tenta com x^3+9.
>>
>> Em dom, 16 de ago de 2020 15:24, Claudio Buffara <
>> claudio.buff...@gmail.com> escreveu:
>>
>>> f(x) em Z[x], bem entendido...
>>>
>>>
>>> On Sun, Aug 16, 2020 at 3:08 PM Claudio Buffara <
>>> claudio.buff...@gmail.com> wrote:
>>>
>>>> Que tal essa aqui?
>>>> Prove ou disprove que, dado um polinômio f(x), irredutível sobre Q,
>>>> existe um inteiro N tal que a irredutibilidade de f pode ser provada pelo
>>>> critério de Eisenstein aplicado a f(x+N).
>>>>
>>>> On Sun, Aug 16, 2020 at 2:31 PM Matheus Secco 
>>>> wrote:
>>>>
>>>>> O melhor jeito é pensar na contrapositiva (supondo que você esteja
>>>>> falando sobre irredutibilidade em Z[x] ou até em Q[x]): se f(x) fatora 
>>>>> como
>>>>> g(x)*h(x), então f(x+a) fatora como g(x+a) *h(x+a) e é claro que uma vez
>>>>> que g(x) e h(x) têm coeficientes inteiros, então g(x+a) e h(x+a) também
>>>>> têm. A recíproca é essencialmente idêntica.
>>>>>
>>>>> Abraços
>>>>>
>>>>> Em dom, 16 de ago de 2020 14:11, Luís Lopes 
>>>>> escreveu:
>>>>>
>>>>>> Sauda,c~oes,
>>>>>>
>>>>>> Como provar que um polinômio f(x) tendo como coeficientes números
>>>>>> inteiros
>>>>>> é irredutível se e somente se f(x+a) é irredutível para algum 
>>>>>> inteiro ?
>>>>>>
>>>>>> Luís
>>>>>>
>>>>>>
>>>>>>
>>>>>>
>>>>>> --
>>>>>> Esta mensagem foi verificada pelo sistema de antivírus e
>>>>>> acredita-se estar livre de perigo.
>>>>>>
>>>>>
>>>>> --
>>>>> Esta mensagem foi verificada pelo sistema de antivírus e
>>>>> acredita-se estar livre de perigo.
>>>>
>>>>
>>> --
>>> Esta mensagem foi verificada pelo sistema de antivírus e
>>> acredita-se estar livre de perigo.
>>
>>
>> --
>> Esta mensagem foi verificada pelo sistema de antivírus e
>> acredita-se estar livre de perigo.
>
>
> --
> Esta mensagem foi verificada pelo sistema de antivírus e
> acredita-se estar livre de perigo.

-- 
Esta mensagem foi verificada pelo sistema de antiv�rus e
 acredita-se estar livre de perigo.



[obm-l] Re: [obm-l] Re: [obm-l] Re: [obm-l] polinômio irredutível

2020-08-16 Por tôpico Esdras Muniz
Tenta com x^3+9.

Em dom, 16 de ago de 2020 15:24, Claudio Buffara 
escreveu:

> f(x) em Z[x], bem entendido...
>
>
> On Sun, Aug 16, 2020 at 3:08 PM Claudio Buffara 
> wrote:
>
>> Que tal essa aqui?
>> Prove ou disprove que, dado um polinômio f(x), irredutível sobre Q,
>> existe um inteiro N tal que a irredutibilidade de f pode ser provada pelo
>> critério de Eisenstein aplicado a f(x+N).
>>
>> On Sun, Aug 16, 2020 at 2:31 PM Matheus Secco 
>> wrote:
>>
>>> O melhor jeito é pensar na contrapositiva (supondo que você esteja
>>> falando sobre irredutibilidade em Z[x] ou até em Q[x]): se f(x) fatora como
>>> g(x)*h(x), então f(x+a) fatora como g(x+a) *h(x+a) e é claro que uma vez
>>> que g(x) e h(x) têm coeficientes inteiros, então g(x+a) e h(x+a) também
>>> têm. A recíproca é essencialmente idêntica.
>>>
>>> Abraços
>>>
>>> Em dom, 16 de ago de 2020 14:11, Luís Lopes 
>>> escreveu:
>>>
 Sauda,c~oes,

 Como provar que um polinômio f(x) tendo como coeficientes números
 inteiros
 é irredutível se e somente se f(x+a) é irredutível para algum 
 inteiro ?

 Luís




 --
 Esta mensagem foi verificada pelo sistema de antivírus e
 acredita-se estar livre de perigo.

>>>
>>> --
>>> Esta mensagem foi verificada pelo sistema de antivírus e
>>> acredita-se estar livre de perigo.
>>
>>
> --
> Esta mensagem foi verificada pelo sistema de antivírus e
> acredita-se estar livre de perigo.

-- 
Esta mensagem foi verificada pelo sistema de antiv�rus e
 acredita-se estar livre de perigo.



[obm-l] Re: [obm-l] Re: [obm-l] Re: [obm-l] Mostrar que está função não existe

2020-08-11 Por tôpico Esdras Muniz
Depois de ver essa solicitação genial, fiquei com vergonha de mandar a
minha.

Em ter, 11 de ago de 2020 01:37, Ralph Costa Teixeira 
escreveu:

> Acho que isso caiu numa IMO que eu fiz Ah, achei, 1987. Aqui tem uma
> resposta bem legal:
>
>
> https://math.stackexchange.com/questions/325504/imo-1987-function-such-that-ffn-n1987
>
>
> On Tue, Aug 11, 2020 at 12:50 AM  wrote:
>
>> É, fatou dizer que k é ímpar
>>
>> Artur
>>
>> Em 10 de ago de 2020 22:33, Ralph Costa Teixeira 
>> escreveu:
>>
>> K inteiro... ímpar? Porque tomando f(n)=n+k/2...
>>
>> On Mon, Aug 10, 2020, 22:05 Artur Costa Steiner <
>> artur.costa.stei...@gmail.com> wrote:
>>
>> Me pareceu que isto era simples, mas segui um caminho errado e ainda não
>> cheguei lá.
>>
>> Mostre que não existe f:N --> N, N os naturais com o 0, tal que f(f(n)) =
>> n + k, k > 0 inteiro.
>>
>> Obrigado
>>
>> Artur
>>
>> --
>> Esta mensagem foi verificada pelo sistema de antivírus e
>> acredita-se estar livre de perigo.
>>
>>
>> --
>> Esta mensagem foi verificada pelo sistema de antiv�rus e
>> acredita-se estar livre de perigo.
>>
>>
>>
>> --
>> Esta mensagem foi verificada pelo sistema de antivírus e
>> acredita-se estar livre de perigo.
>
>
> --
> Esta mensagem foi verificada pelo sistema de antivírus e
> acredita-se estar livre de perigo.

-- 
Esta mensagem foi verificada pelo sistema de antiv�rus e
 acredita-se estar livre de perigo.



[obm-l] Re: [obm-l] Re: [obm-l] Re: [obm-l] Álgebra

2020-08-05 Por tôpico Esdras Muniz
Acho que dá -2. Usa que (x+y)^2=xy e (x/y)^3=1.

Em qua, 5 de ago de 2020 20:07, Anderson Torres <
torres.anderson...@gmail.com> escreveu:

> Em ter., 14 de jul. de 2020 às 23:39, Pacini Bores
>  escreveu:
> >
> > A expressão pedida ao quadrado é igual a 4, sem usar complexos.
> >
> > Pacini
> >
> > Em 14/07/2020 21:50, marcone augusto araújo borges escreveu:
> >
> > Se x^2 +xy + y^2  = 0, com x,y <>0
> > Determinar (x/(x+y))^2019 + (y/(x+y))^2019, sem usar números complexos.
>
> Bem, é meio óbvio que x!=y e x!=-y, senão daria 0.
>
> Podemos supor sem perda de generalidade que x+y=1 (basta dividir x e y
> pela soma)
>
> Assim, temos x+y=1 e x^2+2xy+y^2=1, portanto xy=1.
>
> Assim x e y são zeros do polinômio P(x)=x^2-x+1, e x^2019+y^2019 seria
> calculável mediante uma recorrência.
>
>
> >
> > --
> > Esta mensagem foi verificada pelo sistema de antivírus e
> > acredita-se estar livre de perigo.
> >
> >
> >
> > --
> > Esta mensagem foi verificada pelo sistema de antivírus e
> > acredita-se estar livre de perigo.
>
> --
> Esta mensagem foi verificada pelo sistema de antivírus e
>  acredita-se estar livre de perigo.
>
>
> =
> Instru�ões para entrar na lista, sair da lista e usar a lista em
> http://www.mat.puc-rio.br/~obmlistas/obm-l.html
> =
>

-- 
Esta mensagem foi verificada pelo sistema de antiv�rus e
 acredita-se estar livre de perigo.



[obm-l] Re: [obm-l] Re: [obm-l] Ajuda em teoria dos números

2020-07-24 Por tôpico Esdras Muniz
Se for solução inteira positiva, acho que só tem 3 e 4. Vc supõe spdg x
maior ou igual a y, vê que y=1 não tem solução e x=y tb não. Daí, x>y>1.
Fatorando a expressão, fica: (xy-8-(x-y))(xy-8+(x-y))=15. Como
(xy-8-(x-y))>(xy-8+(x-y))>-2.
Temos que ou (xy-8-(x-y))=1 e (xy-8+(x-y))=15, o que não tem soluções
inteiras positivas, ou (xy-8-(x-y))=3 e (xy-8+(x-y))=5, cujas únicas
soluções inteiras são x=4 e y=3.

Em sex, 24 de jul de 2020 10:36, Claudio Buffara 
escreveu:

> Pelo que entendi, a solução é a porção dessa curva algébrica situada no 1o
> quadrante.
> Dá pra fazer isso no Wolfram Alpha, com o comando plot (x*y-7)^2 - x^2 -
> y^2 = 0.
>
> []s,
> Claudio.
>
> On Fri, Jul 24, 2020 at 9:58 AM Prof. Douglas Oliveira <
> profdouglaso.del...@gmail.com> wrote:
>
>> Preciso de ajuda para encontrar todas as soluções não negativas da equação
>> (xy-7)^2=x^2+y^2.
>>
>> Desde já agradeço a ajuda
>> Douglas Oliveira
>>
>> --
>> Esta mensagem foi verificada pelo sistema de antivírus e
>> acredita-se estar livre de perigo.
>
>
> --
> Esta mensagem foi verificada pelo sistema de antivírus e
> acredita-se estar livre de perigo.

-- 
Esta mensagem foi verificada pelo sistema de antiv�rus e
 acredita-se estar livre de perigo.



[obm-l] Re: [obm-l] Re: [obm-l] Re: [obm-l] Re: [obm-l] Re: [obm-l] Re: [obm-l] Questão OBM - U

2020-01-23 Por tôpico Esdras Muniz
É fácil ver que esse ínfimo tem que ser no mínimo 4, basta fazer
desigualdade triângulos com os triângulos que têm dois vértices comuns com
o quadrilátero e o terceiro sendo a interseção das diagonais. E por esse
argumento do Caio, vemos que é 4 mesmo.

Em qui, 23 de jan de 2020 08:59, Caio Costa  escreveu:

> Minimiza-se a soma das diagonais ao tomar-se um losango degenerado, com
> uma diagonal valendo 4 e outra valendo 0.
>
> Em qui, 23 de jan de 2020 08:34, gilberto azevedo 
> escreveu:
>
>> Pensei em minimizar √(a² + (4-a)²)
>> 4 - a, devido ao fato do perímetro ser 8.
>> No caso obtenho o mínimo sendo 2√2, quando o retângulo é um quadrado de
>> lado 2.
>> A soma das diagonais seria no caso 4√2, e não bate com o gabarito.
>>
>> Em qui, 23 de jan de 2020 08:20, Bernardo Freitas Paulo da Costa <
>> bernardo...@gmail.com> escreveu:
>>
>>> On Thu, Jan 23, 2020 at 7:24 AM gilberto azevedo 
>>> wrote:
>>> >> On Sat, Jan 11, 2020 at 11:24 AM gilberto azevedo <
>>> gil159...@gmail.com> wrote:
>>> >> >
>>> >> > Qual o ínfimo sobre todos os quadriláteros convexos com
>>> perímetro 8 da soma dos comprimentos de suas diagonais ?
>>> >
>>> > Tentei com o retângulo e o quadrado, porém não obtive a resposta...  O
>>> gabarito é 4.
>>>
>>> Qual (ou quais?) retângulo(s) você testou??  Que resposta você obteve?
>>> --
>>> Bernardo Freitas Paulo da Costa
>>>
>>> --
>>> Esta mensagem foi verificada pelo sistema de antivírus e
>>>  acredita-se estar livre de perigo.
>>>
>>>
>>> =
>>> Instru�ões para entrar na lista, sair da lista e usar a lista em
>>> http://www.mat.puc-rio.br/~obmlistas/obm-l.html
>>> =
>>>
>>
>> --
>> Esta mensagem foi verificada pelo sistema de antivírus e
>> acredita-se estar livre de perigo.
>
>
> --
> Esta mensagem foi verificada pelo sistema de antivírus e
> acredita-se estar livre de perigo.

-- 
Esta mensagem foi verificada pelo sistema de antiv�rus e
 acredita-se estar livre de perigo.



Re: [obm-l] Soma de Riemann

2020-01-13 Por tôpico Esdras Muniz
Esse limite vai ser a integral inferior de sen(x) de 0 a b. Daí, como Sen é
integravel, esse limite vai ser Sen(b).

Em dom, 12 de jan de 2020 19:19, Luiz Antonio Rodrigues <
rodrigue...@gmail.com> escreveu:

> Olá, pessoal!
> Tudo bem?
> Estou pensando neste problema há vários dias e não consigo descobrir onde
> está meu erro.
> Alguém pode me ajudar?
>
> O problema é o seguinte:
>
> É dado o somatório de:
>
> sen(k*b/n)
>
> Onde k varia de 1 até n.
>
> Preciso calcular o limite deste somatório dividido por n, quando n tende a
> infinito.
>
> O problema pede que se relacione este limite com uma soma de Riemann.
>
> Eu cheguei no valor zero, que está errado.
> O problema parece simples...
> Agradeço desde já!
> Luiz
>
>
> --
> Esta mensagem foi verificada pelo sistema de antivírus e
> acredita-se estar livre de perigo.

-- 
Esta mensagem foi verificada pelo sistema de antiv�rus e
 acredita-se estar livre de perigo.



[obm-l] Re: [obm-l] Potência

2020-01-11 Por tôpico Esdras Muniz
Acho que é d) 04

Em sáb, 11 de jan de 2020 11:01, Esdras Muniz 
escreveu:

> Pode usar a função fi.
>
> Em sáb, 11 de jan de 2020 10:23, Vanderlei Nemitz 
> escreveu:
>
>> Bom dia!
>> Eu resolvi essa questão, mas creio que trabalhei demais!
>>
>> Alguém conhece um modo relativamente simples?
>>
>> Os dois últimos algarismos de 2^222 são:
>> a) 84
>> b) 24
>> c) 64
>> d) 04
>> e) 44
>>
>> Muito obrigado!
>>
>> Vanderlei
>>
>> --
>> Esta mensagem foi verificada pelo sistema de antivírus e
>> acredita-se estar livre de perigo.
>
>

-- 
Esta mensagem foi verificada pelo sistema de antiv�rus e
 acredita-se estar livre de perigo.



[obm-l] Re: [obm-l] Potência

2020-01-11 Por tôpico Esdras Muniz
Pode usar a função fi.

Em sáb, 11 de jan de 2020 10:23, Vanderlei Nemitz 
escreveu:

> Bom dia!
> Eu resolvi essa questão, mas creio que trabalhei demais!
>
> Alguém conhece um modo relativamente simples?
>
> Os dois últimos algarismos de 2^222 são:
> a) 84
> b) 24
> c) 64
> d) 04
> e) 44
>
> Muito obrigado!
>
> Vanderlei
>
> --
> Esta mensagem foi verificada pelo sistema de antivírus e
> acredita-se estar livre de perigo.

-- 
Esta mensagem foi verificada pelo sistema de antiv�rus e
 acredita-se estar livre de perigo.



[obm-l] Re: [obm-l] Função Desconhecida

2019-12-20 Por tôpico Esdras Muniz
Acho que essa função é trancendente.

Em sex, 20 de dez de 2019 14:42, Luiz Antonio Rodrigues <
rodrigue...@gmail.com> escreveu:

> Olá, pessoal!
> Tudo bem?
> Estou tentando, há alguns dias, resolver o seguinte problema:
>
> Preciso descobrir uma função f(x) cuja derivada é sen(x^3). Sabe-se que
> f(0)=2.
>
> Utilizei um software e mesmo assim não cheguei numa resposta para esta
> integral...
> Alguém sabe se esta função é de algum tipo "especial"?
> Muito obrigado!
> Luiz
>
> --
> Esta mensagem foi verificada pelo sistema de antivírus e
> acredita-se estar livre de perigo.

-- 
Esta mensagem foi verificada pelo sistema de antiv�rus e
 acredita-se estar livre de perigo.



Re: [obm-l] soma com cevianas que passam pelo circuncentro

2019-12-19 Por tôpico Esdras Muniz
Eu tinha feito algo parecido com essa prova 2. Usando o método k.

Em qui, 19 de dez de 2019 14:43, Luís Lopes 
escreveu:

> Sauda,c~oes,
>
> Encontrei um link com a prova:
>
> https://www.cut-the-knot.org/m/Geometry/CeviansThroughCircumcenter.shtml
>
> Esse site é muito bom.
>
> Eu conhecia a prova 3 mas não sabia que o triângulo tinha que ser
> acutângulo.
> Para triângulo retângulo vale também, por verificação direta.
>
> Aí comecei a rever a prova para triângulos obtusângulos e vi que
> havia um problema com (B-C)=90º. Acho que para triângulos obtusângulos
> a igualdade pode valer mas tem que ver para quais casos ela
> não serve. Talvez (B-C) > 90º como (115º,15º,50º) e (B-C) < 90º
> como (105º,45º,30º) satisfazem mas (B-C) = 90º como (120º,30º,30º)
> não satisfaz. Isso precisaria de outra investigação.
>
> Abraços,
> Luís
>
>
>
>
>
> --
> Esta mensagem foi verificada pelo sistema de antivírus e
> acredita-se estar livre de perigo.
>

-- 
Esta mensagem foi verificada pelo sistema de antiv�rus e
 acredita-se estar livre de perigo.



[obm-l] Re: [obm-l] Teoria dos números

2019-12-13 Por tôpico Esdras Muniz
Existe congruência com números que não são inteiros?

Em sex, 13 de dez de 2019 11:57, Prof. Douglas Oliveira <
profdouglaso.del...@gmail.com> escreveu:

> Olá caros amigos,
> preciso de uma ajuda pra criar uma fórmula que seja congruente (módulo p)
> ao somatório
> S_a=sum{(a^k)/k}, com k de 1 a p-1, sendo p primo ímpar.
>
> Saudações
> Douglas Oliveira
>
> --
> Esta mensagem foi verificada pelo sistema de antivírus e
> acredita-se estar livre de perigo.

-- 
Esta mensagem foi verificada pelo sistema de antiv�rus e
 acredita-se estar livre de perigo.



Re: [obm-l]

2019-12-12 Por tôpico Esdras Muniz
Isso aí pode ir para o infinito: tome k real positivo arbitrário. Daí tome:
(-k)+(-k)+...+(-k)+(n-1)k=0
(-k)^3+(-k)^3+...+(-k)^3+((n-1)k)^3=k^3((n-1)^3-(n-1)).
Esse último fator vai pra o infinito com k.

Em qui, 12 de dez de 2019 18:20, Anderson Torres <
torres.anderson...@gmail.com> escreveu:

> Suas postagens vêm sempre sem título?
>
> Em seg., 9 de dez. de 2019 às 20:29, gilberto azevedo
>  escreveu:
> >
> > Sabendo que :
> > x_1 + ... + x_n = 0
> > x_1 ² + ... + x_n ² = 1
> > Qual o valor máximo de x_1 ³ + ... + x_n ³ ?
> >
> > --
> > Esta mensagem foi verificada pelo sistema de antivírus e
> > acredita-se estar livre de perigo.
>
> --
> Esta mensagem foi verificada pelo sistema de antivírus e
>  acredita-se estar livre de perigo.
>
>
> =
> Instru�ões para entrar na lista, sair da lista e usar a lista em
> http://www.mat.puc-rio.br/~obmlistas/obm-l.html
> =
>

-- 
Esta mensagem foi verificada pelo sistema de antiv�rus e
 acredita-se estar livre de perigo.



Re: [obm-l] Funcional equation

2019-12-09 Por tôpico Esdras Muniz
Vi um jeito de mostrar que só tem no máximo uma solução com grau n para
cada n.

Em ter, 10 de dez de 2019 00:11, Pedro Cardoso 
escreveu:

> Minha intuição foi a seguinte, considere a sequência a_0=0 e
> a_(n+1)=(a_n)²+1
>
> Agora pomos P(0)=c
> Pela equação funcional, P(0²+1)=c²+1
> E P((0²+1)²+1)=(c²+1)²+1
>
> Em geral, se f(x)=x²+1, então
>
> P(a_n)=fⁿ(c), em que fⁿ é a iteração de f n vezes.
>
> Assim, se c=a_n para algum m natural, então vamos ter
>
> P(a_0)=a_n
> P(a_1)=a_(n+1)
>
> E em geral, P(x)=fⁿ(x)
>
> Ainda não consegui provar que P não vai ser um polinômio se c não for
> algum a_n
>
> Em seg, 9 de dez de 2019 22:43, Pedro Cardoso 
> escreveu:
>
>> Preciso pensar mais, mas suspeito que seja qualquer polinômio do tipo
>>
>> (...((x²+1)²+1)²...)²+1
>>
>> Os primeiros são
>>
>> x
>> x²+1
>> x⁴+2x²+2
>> ...
>>
>
> --
> Esta mensagem foi verificada pelo sistema de antivírus e
> acredita-se estar livre de perigo.

-- 
Esta mensagem foi verificada pelo sistema de antiv�rus e
 acredita-se estar livre de perigo.



Re: [obm-l]

2019-11-29 Por tôpico Esdras Muniz
Tentei fazer o mesmo com R=1e l=√3, mas desisti qdo vi o tamanho das contas.

Em sex, 29 de nov de 2019 16:09, Claudio Buffara 
escreveu:

> Acho que com números complexos e alguma álgebra sai.
>
> Se os vértices do triângulo forem R, Rw  e Rw^2 (onde w = cis(2pi/3) e R é
> um real positivo) e P = z, então:
> a = |z - R|, b = |z - Rw|; c = |z - Rw^2| ==>
> a^2 + b^2 + c^2 = |z - R|^2 + |z - Rw|^2 + |z - Rw^2|^2 = 3*|z|^2 + 3*R^2
>  (se não errei nenhuma conta)
>
> Neste caso, L^2 = 3*R^2, de modo que o lado direito da expressão do
> enunciado será igual a (3*|z|^2 + 6*R^2)^2 = 9*(|z|^4 + 4*R^2*|z|^2 +
> 4*R^4).
>
> O lado esquerdo deve dar um pouco mais de trabalho...
>
>
> On Tue, Nov 26, 2019 at 7:00 PM gilberto azevedo 
> wrote:
>
>> Pesquisando achei uma relação muito interessante, mas não achei nenhuma
>> demonstração dela na web.
>> Pra quem se interessar Seja um ponto P no interior de um triângulo
>> equilátero de lado l, e a,b,c a distância desse ponto aos vértices do
>> triângulo. Provar que :
>> 3( a⁴ + b⁴ + c⁴ + l⁴) = ( a² + b² + c² + l²)²
>>
>> --
>> Esta mensagem foi verificada pelo sistema de antivírus e
>> acredita-se estar livre de perigo.
>
>
> --
> Esta mensagem foi verificada pelo sistema de antivírus e
> acredita-se estar livre de perigo.

-- 
Esta mensagem foi verificada pelo sistema de antiv�rus e
 acredita-se estar livre de perigo.



Re: [obm-l]

2019-11-28 Por tôpico Esdras Muniz
Para cada i de 1 a 2019, n= k+k+1+...+k+i=(i+1)×k+t_i onde t_i =i(i+1)/2.
Daí, i+1|n se i for ímpar ou (i+1)/2 |n se i for par. Daí peguei o produto
dos ímpares vezes a maior potência de 2 menor que 2019.

Percebi agora que deve dar certo com mmc(1, 3, ..., 2019).

Em qui, 28 de nov de 2019 19:57, Jamil Silva 
escreveu:

> Qual o raciocínio que leva a esse resultado ?
>
>
>
> Enviado do Email <https://go.microsoft.com/fwlink/?LinkId=550986> para
> Windows 10
>
>
> --
> *De:* owner-ob...@mat.puc-rio.br  em nome de
> Esdras Muniz 
> *Enviado:* Thursday, November 28, 2019 6:18:00 PM
> *Para:* obm-l@mat.puc-rio.br 
> *Assunto:* Re: [obm-l]
>
> Acho que é (2019!)/(2^{1000}×1009!).
>
> Em qui, 28 de nov de 2019 12:41, Jamil Silva 
> escreveu:
>
> Qual o menor número que  possui exatamente 2019 partições tal que em todas
> elas as partes sejam números inteiros positivos e consecutivos ?
>
> --
> Esta mensagem foi verificada pelo sistema de antivírus e
> acredita-se estar livre de perigo.
>
>
> --
> Esta mensagem foi verificada pelo sistema de antiv�rus e
> acredita-se estar livre de perigo.
>
> --
> Esta mensagem foi verificada pelo sistema de antivírus e
> acredita-se estar livre de perigo.
>

-- 
Esta mensagem foi verificada pelo sistema de antiv�rus e
 acredita-se estar livre de perigo.



Re: [obm-l]

2019-11-28 Por tôpico Esdras Muniz
Acho que é (2019!)/(2^{1000}×1009!).

Em qui, 28 de nov de 2019 12:41, Jamil Silva 
escreveu:

> Qual o menor número que  possui exatamente 2019 partições tal que em todas
> elas as partes sejam números inteiros positivos e consecutivos ?
>
> --
> Esta mensagem foi verificada pelo sistema de antivírus e
> acredita-se estar livre de perigo.
>

-- 
Esta mensagem foi verificada pelo sistema de antiv�rus e
 acredita-se estar livre de perigo.



Re: [obm-l] A c o i s a f i c o u f e i a e m 2 0 1 9

2019-11-28 Por tôpico Esdras Muniz
Primeiro, troque os hóspedes que já estão no hotel de quarto, mandando o
hóspede do quarto n para o quarto 2n, assim, todos os quartos ímpares
estarão desocupados.
Depois, faça uma bijeção entre os ônibus e os naturais {1, 2, 3, ...}. Em
seguida, faça uma bijeção entre os hóspedes do n-esimo ônibus e os
racionais no intervalo [n-1, n).

Por fim, é só tomar uma bijeção entre os racionais não negativos e os
números ímpares.

Em qui, 28 de nov de 2019 08:05, Jamil Silva 
escreveu:

> Joãozinho passou o ano desempregado mas conseguiu agora na alta temporada
> um emprego de gerente no hotel Georg Cantor.
> Logo no seu primeiro dia, pela manhã, ele fica sabendo que todos os
> quartos já estão ocupados e que ao final do dia chegarão
> infinitos ônibus, cada um com infinitos hóspedes.
>
> O Georg Cantor é famoso por ter infinitos quartos e ser rigoroso em sua
> politica de acomodar um e apenas um hóspede por quarto,
>  devendo o gerente  manter todos os quartos ocupados, seguindo,
> rigorosamente, essa política.
>
> Será possível, nas condições acima, acomodar cada um dos infinitos
> passageiros de cada um dos infinitos ônibus no hotel já lotado ?
>
> --
> Esta mensagem foi verificada pelo sistema de antivírus e
>  acredita-se estar livre de perigo.
>
> =
> Instru�ões para entrar na lista, sair da lista e usar a lista em
> http://www.mat.puc-rio.br/~obmlistas/obm-l.html
> =
>

-- 
Esta mensagem foi verificada pelo sistema de antiv�rus e
 acredita-se estar livre de perigo.



Re: [obm-l] Quadrados perfeitos

2019-11-27 Por tôpico Esdras Muniz
Percebi agora que tô errado. Desculpa.

Em qua, 27 de nov de 2019 19:22, Esdras Muniz 
escreveu:

> Pensei assim, o 10^10= (10^5)^2 é qp, daí, (10^5+1)^2, (10^5+2)^2, ...,
> [Sqrt{12×10^5}] são só quadrados que queremos contar.
>
> Estou usando [x] para demorar a parte interna de x.
>
> Em qua, 27 de nov de 2019 15:30, Caio Costa 
> escreveu:
>
>> 10^5([sqrt{2}]-1) ??
>>
>>
>> Em qua., 27 de nov. de 2019 às 13:41, Esdras Muniz <
>> esdrasmunizm...@gmail.com> escreveu:
>>
>>> 10^5([sqrt{12}]-1)
>>>
>>> Em qua, 27 de nov de 2019 08:57, marcone augusto araújo borges <
>>> marconeborge...@hotmail.com> escreveu:
>>>
>>>> Seja n E N tal que 1 < = n < = 10^10. Quantos  números M = 11n + 10^10
>>>> são quadrados perfeitos?
>>>> --
>>>> Esta mensagem foi verificada pelo sistema de antivírus e
>>>> acredita-se estar livre de perigo.
>>>>
>>>
>>> --
>>> Esta mensagem foi verificada pelo sistema de antivírus e
>>> acredita-se estar livre de perigo.
>>
>>
>> --
>> Esta mensagem foi verificada pelo sistema de antivírus e
>> acredita-se estar livre de perigo.
>
>

-- 
Esta mensagem foi verificada pelo sistema de antiv�rus e
 acredita-se estar livre de perigo.



Re: [obm-l] Quadrados perfeitos

2019-11-27 Por tôpico Esdras Muniz
Pensei assim, o 10^10= (10^5)^2 é qp, daí, (10^5+1)^2, (10^5+2)^2, ...,
[Sqrt{12×10^5}] são só quadrados que queremos contar.

Estou usando [x] para demorar a parte interna de x.

Em qua, 27 de nov de 2019 15:30, Caio Costa  escreveu:

> 10^5([sqrt{2}]-1) ??
>
>
> Em qua., 27 de nov. de 2019 às 13:41, Esdras Muniz <
> esdrasmunizm...@gmail.com> escreveu:
>
>> 10^5([sqrt{12}]-1)
>>
>> Em qua, 27 de nov de 2019 08:57, marcone augusto araújo borges <
>> marconeborge...@hotmail.com> escreveu:
>>
>>> Seja n E N tal que 1 < = n < = 10^10. Quantos  números M = 11n + 10^10
>>> são quadrados perfeitos?
>>> --
>>> Esta mensagem foi verificada pelo sistema de antivírus e
>>> acredita-se estar livre de perigo.
>>>
>>
>> --
>> Esta mensagem foi verificada pelo sistema de antivírus e
>> acredita-se estar livre de perigo.
>
>
> --
> Esta mensagem foi verificada pelo sistema de antivírus e
> acredita-se estar livre de perigo.

-- 
Esta mensagem foi verificada pelo sistema de antiv�rus e
 acredita-se estar livre de perigo.



Re: [obm-l] Quadrados perfeitos

2019-11-27 Por tôpico Esdras Muniz
10^5([sqrt{12}]-1)

Em qua, 27 de nov de 2019 08:57, marcone augusto araújo borges <
marconeborge...@hotmail.com> escreveu:

> Seja n E N tal que 1 < = n < = 10^10. Quantos  números M = 11n + 10^10
> são quadrados perfeitos?
> --
> Esta mensagem foi verificada pelo sistema de antivírus e
> acredita-se estar livre de perigo.
>

-- 
Esta mensagem foi verificada pelo sistema de antiv�rus e
 acredita-se estar livre de perigo.



[obm-l] Re: [obm-l] Re: [obm-l] Re: [obm-l] Triângulos.

2019-11-24 Por tôpico Esdras Muniz
Verdade, não tinha percebido.

Em dom, 24 de nov de 2019 14:17, Pedro José  escreveu:

> Boa tarde!
> Esdras,
> Não seria z>=3.
> 3, 2, 2 dá um obtusângulo.
>
> Saudações,
> PJMS
>
> Em sáb, 23 de nov de 2019 01:52, Esdras Muniz 
> escreveu:
>
>> Acho que a questão pressupõe que os lados devem ser inteiros. Daí se os
>> lados são x, y e z, com x<=y> x^2+y^2x^2+y^2 e
>> z> Daí, z é ao menos 4, vc sai contando caso a caso...
>>
>> Em sex, 22 de nov de 2019 20:39, Claudio Buffara <
>> claudio.buff...@gmail.com> escreveu:
>>
>>> Do jeito que está escrito, uma infinidade.
>>>
>>> Enviado do meu iPhone
>>>
>>> > Em 22 de nov de 2019, à(s) 19:18, Guilherme Abbehusen <
>>> gui.abbehuse...@gmail.com> escreveu:
>>> >
>>> > 
>>> > Olá,Â
>>> >   Preciso de ajuda com a seguinte questão:Â
>>> >
>>> > Tendo em vista a leis dos Cossenos, marque a quantidade de triângulos
>>> obtusângulos que podemos formar com lados menores do que 7.
>>> > a) 6
>>> > b) 7
>>> > c) 8Â
>>> > d) 9
>>> > e) 10
>>> >
>>> > --
>>> > Esta mensagem foi verificada pelo sistema de antivírus e
>>> > acredita-se estar livre de perigo.
>>>
>>> --
>>> Esta mensagem foi verificada pelo sistema de antivírus e
>>>  acredita-se estar livre de perigo.
>>>
>>>
>>> =
>>> Instru�ões para entrar na lista, sair da lista e usar a lista em
>>> http://www.mat.puc-rio.br/~obmlistas/obm-l.html
>>> =
>>>
>>
>> --
>> Esta mensagem foi verificada pelo sistema de antivírus e
>> acredita-se estar livre de perigo.
>
>
> --
> Esta mensagem foi verificada pelo sistema de antivírus e
> acredita-se estar livre de perigo.

-- 
Esta mensagem foi verificada pelo sistema de antiv�rus e
 acredita-se estar livre de perigo.



[obm-l] Re: [obm-l] Triângulos.

2019-11-22 Por tôpico Esdras Muniz
Acho que a questão pressupõe que os lados devem ser inteiros. Daí se os
lados são x, y e z, com x<=yx^2+y^2 e
z
escreveu:

> Do jeito que está escrito, uma infinidade.
>
> Enviado do meu iPhone
>
> > Em 22 de nov de 2019, à(s) 19:18, Guilherme Abbehusen <
> gui.abbehuse...@gmail.com> escreveu:
> >
> > 
> > Olá,Â
> >   Preciso de ajuda com a seguinte questão:Â
> >
> > Tendo em vista a leis dos Cossenos, marque a quantidade de triângulos
> obtusângulos que podemos formar com lados menores do que 7.
> > a) 6
> > b) 7
> > c) 8Â
> > d) 9
> > e) 10
> >
> > --
> > Esta mensagem foi verificada pelo sistema de antivírus e
> > acredita-se estar livre de perigo.
>
> --
> Esta mensagem foi verificada pelo sistema de antivírus e
>  acredita-se estar livre de perigo.
>
>
> =
> Instru�ões para entrar na lista, sair da lista e usar a lista em
> http://www.mat.puc-rio.br/~obmlistas/obm-l.html
> =
>

-- 
Esta mensagem foi verificada pelo sistema de antiv�rus e
 acredita-se estar livre de perigo.



Re: [obm-l]

2019-11-22 Por tôpico Esdras Muniz
Eu usei mg>= mh

Em sex, 22 de nov de 2019 17:04, Claudio Buffara 
escreveu:

> Que podemos elevar ao quadrado, obtendo x^6/(x - 12).
>
> Ou seja, o problema se torna achar o valor mínimo de x^6/(x - 12), com x >
> 12  (não pode ser "=" ...).
> Depois, é só tirar a raiz quadrada.
>
> Agora, usamos a sugestão do Julio: y^6 = x - 12 ==> x^6 = (y^6 + 12)^6.
> E a expressão a ser minimizada passa a ser (y^6 + 12)^6/y^6 = ((y^6 +
> 12)/y)^6 = (y^5 + 12/y)^6.
>
> y^5 + 12/y = 6*(y^5 + 5*(2,4/y))/6 >= 6*(y^5*(2,4/y)^5)^(1/6)  (MA >= MG)
> = 6*2,4^(5/6).
>
> Ou seja, o valor mínimo de y^5 + 12/y é igual a 6*(12/5)^(5/6), e é obtido
> quando y^5 = 2,4/y <==> y = (12/5)^(1/6).
> ==> y^6 = 12/5 = x - 12
> ==> x = 12 + 12/5 = 72/5
> ==> x^6/(x-12) = (72/5)^6/(12/5) = 6^6*12^5/5^5 = 2^16*3^11/5^5
> ==> o valor mínimo de raiz(x^6/(x-12)) é igual a 2^8*3^(11/2)/5^(5/2) e é
> atingido quando x = 72/5.
>
> Moral da história: se souber usar derivada, use derivada...
>
> []s,
> Claudio.
>
> On Fri, Nov 22, 2019 at 4:22 PM Claudio Buffara 
> wrote:
>
>> Melhor reescrever a expressão.
>> Como x - 12 >= 0, podemos supor que x >= 12.
>> Nesse caso, a expressão a ser minimizada fica x^3/raiz(x-12), certo?
>>
>> On Fri, Nov 22, 2019 at 4:20 PM gilberto azevedo 
>> wrote:
>>
>>> Não vejo com isso ajuda. Eu tava pensando em usa AM - MG , mas n ajudou
>>> mt.
>>>
>>> Em sex, 22 de nov de 2019 10:10, Julio César Saldaña Pumarica <
>>> saldana...@pucp.edu.pe> escreveu:
>>>
 mudando a variável:

 x-12 = y^6

 El vie., 22 nov. 2019 a las 2:40, gilberto azevedo (<
 gil159...@gmail.com>) escribió:

> Como achar o mínimo de :
> x² * √(x²/(x-12)) , usando apenas desigualdades comuns ?
>
> --
> Esta mensagem foi verificada pelo sistema de antivírus e
> acredita-se estar livre de perigo.


 --
 Esta mensagem foi verificada pelo sistema de antivírus e
 acredita-se estar livre de perigo.
>>>
>>>
>>> --
>>> Esta mensagem foi verificada pelo sistema de antivírus e
>>> acredita-se estar livre de perigo.
>>
>>
> --
> Esta mensagem foi verificada pelo sistema de antivírus e
> acredita-se estar livre de perigo.

-- 
Esta mensagem foi verificada pelo sistema de antiv�rus e
 acredita-se estar livre de perigo.



[obm-l] Re: [obm-l] Re: [obm-l] Qual o 2020º termo da sequência abaixo ?

2019-11-18 Por tôpico Esdras Muniz
Eu resolvi fazendo um programa, e deu 17. Mas a ideia é essa mesmo do mod
41. Se aparecerem dois números seguidos que já apareceram antes, a
sequência começar a se repetir, tipo 1, 2,..., 1, 2,... E isso com certeza
vai ocorrer, pois só há 41×40 duplas de números seguidos possíveis,
considerando a ordem. Pelo que vi, a sequência começar a repetir a partir
do termo 60.

Em dom, 17 de nov de 2019 21:54, Claudio Buffara 
escreveu:

> Pela definição da sequência.
> Quando a a(n) + a(n+1) > 40, a(n+2) = resto da divisão de a(n) + a(n+1)
> por 40, sendo que neste caso os restos vão de 1 a 40 (ao invés de 0 a 39).
>
> Enviado do meu iPhone
>
> > Em 17 de nov de 2019, à(s) 18:59, Jamil Silva 
> escreveu:
> >
> > Por que mod40 ?
> >
> > 17.11.2019, 14:36, "Claudio Buffara" :
> >> Me parece que basta calcular o 2020o termo sem a restrição de ser mod
> 40 (é uma sequência de Fibonacci começando por 5 e 2) e depois ver
> quanto e’ a(2020) mod 40, sendo que na redução mod 40, ao invés dos
> restos serem 0, 1, ..., 39, eles serão 1, 2, ..., 40.
> >>
> >> Enviado do meu iPhone
> >>
> >>> Â Em 17 de nov de 2019, Ã (s) 08:15, Jamil Silva <
> jamilsi...@yandex.com> escreveu:
> >>>
> >>>  5, 2, 7, 9, 16, 25, 1, 26, 27, 13, 40, 13, 13, 26, 39, 25, 24,...
> >>>
> >>>  Sua lei de formação é a seguinte:
> >>>
> >>> Â a(1) = 5
> >>> Â a(2) = 2
> >>>  a(n+2) = [a(n+1)+a(n)], sse [a(n+1) + a(n)] ≤ 40
> >>> Â a(n+2) = [a(n+1)+a(n)] - 40, sse [a(n+1) + a(n)] > 40
> >>>
> >>> Â --
> >>>  Esta mensagem foi verificada pelo sistema de antivírus e
> >>> Â acredita-se estar livre de perigo.
> >>>
> >>> Â
> =
> >>>  Instruções para entrar na lista, sair da lista e usar a lista em
> >>> Â http://www.mat.puc-rio.br/~obmlistas/obm-l.html
> >>> Â
> =
> >>
> >> --
> >> Esta mensagem foi verificada pelo sistema de antiv?rus e
> >> Â acredita-se estar livre de perigo.
> >>
> >>
> =
> >> Instru??es para entrar na lista, sair da lista e usar a lista em
> >> http://www.mat.puc-rio.br/~obmlistas/obm-l.html
> >>
> =
> >
> > --
> > Esta mensagem foi verificada pelo sistema de antivírus e
> > acredita-se estar livre de perigo.
> >
> > =
> > Instruções para entrar na lista, sair da lista e usar a lista em
> > http://www.mat.puc-rio.br/~obmlistas/obm-l.html
> > =
>
> --
> Esta mensagem foi verificada pelo sistema de antivírus e
>  acredita-se estar livre de perigo.
>
>
> =
> Instru�ões para entrar na lista, sair da lista e usar a lista em
> http://www.mat.puc-rio.br/~obmlistas/obm-l.html
> =
>

-- 
Esta mensagem foi verificada pelo sistema de antiv�rus e
 acredita-se estar livre de perigo.



[obm-l] Re: [obm-l] Re: [obm-l] Qual o 2020º termo da sequência abaixo ?

2019-11-17 Por tôpico Esdras Muniz
17

Em dom, 17 de nov de 2019 20:59, Jamil Silva 
escreveu:

> Por que mod40 ?
>
> 17.11.2019, 14:36, "Claudio Buffara" :
> > Me parece que basta calcular o 2020o termo sem a restrição de ser mod 40
> (é uma sequência de Fibonacci começando por 5 e 2) e depois ver quanto e’
> a(2020) mod 40, sendo que na redução mod 40, ao invés dos restos serem 0,
> 1, ..., 39, eles serão 1, 2, ..., 40.
> >
> > Enviado do meu iPhone
> >
> >>  Em 17 de nov de 2019, à(s) 08:15, Jamil Silva 
> escreveu:
> >>
> >>  5, 2, 7, 9, 16, 25, 1, 26, 27, 13, 40, 13, 13, 26, 39, 25, 24,...
> >>
> >>  Sua lei de formação é a seguinte:
> >>
> >>  a(1) = 5
> >>  a(2) = 2
> >>  a(n+2) = [a(n+1)+a(n)], sse [a(n+1) + a(n)] ≤ 40
> >>  a(n+2) = [a(n+1)+a(n)] - 40, sse [a(n+1) + a(n)] > 40
> >>
> >>  --
> >>  Esta mensagem foi verificada pelo sistema de antivírus e
> >>  acredita-se estar livre de perigo.
> >>
> >>
>  =
> >>  Instruções para entrar na lista, sair da lista e usar a lista em
> >>  http://www.mat.puc-rio.br/~obmlistas/obm-l.html
> >>
>  =
> >
> > --
> > Esta mensagem foi verificada pelo sistema de antiv?rus e
> >  acredita-se estar livre de perigo.
> >
> > =
> > Instru??es para entrar na lista, sair da lista e usar a lista em
> > http://www.mat.puc-rio.br/~obmlistas/obm-l.html
> > =
>
> --
> Esta mensagem foi verificada pelo sistema de antivírus e
>  acredita-se estar livre de perigo.
>
> =
> Instru�ões para entrar na lista, sair da lista e usar a lista em
> http://www.mat.puc-rio.br/~obmlistas/obm-l.html
> =
>

-- 
Esta mensagem foi verificada pelo sistema de antiv�rus e
 acredita-se estar livre de perigo.



[obm-l] Re: [obm-l] Re: [obm-l] CONTAGEM, SISTEMA DE NUMERAÇÂO E ORDENAÇÂO DE CONJUNTOS

2019-11-13 Por tôpico Esdras Muniz
Tá virando moda esse tipo de problema, já são ao menos 3 parecidos que o
povo coloca aqui. Tem algum artigo ou livro pra estudar esse tipo de
problema?

Em qua, 13 de nov de 2019 16:24, Jamil Silva 
escreveu:

> Só esqueci de dizer que as sequencias são impressas seguindo rigorosamente
> a ordem alfabética de forma contínua, ou seja, sem espaço entre duas
> quaisquer, passando imediatamente de uma para a pagina seguinte em um
> determinado livro e por sua vez entre, da mesma forma, da ultima pagina de
> um livro para a primeira pagina do livro seguinte. os livros são indexados
> ou intitulados cada um com a primeira sequencia impressa nele, mesmo que
> esteja incompleta, ou seja, que tenha começado no livro anterior.
>
> como exemplo, eis as quinze primeiras sequencias com as cem letras da
> primeira linha da primeira pagina do livro A :
>
>
> AaaAAAAaaAAAAaaABacADaeA
> F
>
>
> Note que como cada linha só comporta no máximo cem letras, a ultima das 15
> primeiras sequencias termina na linha seguinte: F
>
> 13.11.2019, 15:41, "Jamil Silva" :
> > Imagine uma enciclopédia contendo todas as sequencias(combinações) das
> vinte e seis letras do alfabeto latino . As sequências têm, no mínimo, uma
> e, no máximo, dez letras. São impressas em
> > páginas de cem linhas e em cada linha há exatamente cem letras. Para se
> distinguir as sequências uma da outra, a impressão é feita em cores preto e
> vermelho, tal que duas sequencias consecutivas quaisquer nunca sejam da
> mesma cor. Cada livro da enciclopédia tem exatamente mil páginas e é
> indexado ou intitulado com a sua primeira sequencia.
> > Determine, com base nestas informações
> >
> > 1) Quantos livros tem essa enciclopédia ?
> > 2) Quantas páginas em branco há no ultimo livro ?
> > 3) Quantas sequencias começam num livro e terminam no seguinte ?
> > 4) Qual a localização precisa da sequência MATEMATICA em termos de
> (linha, página, livro) ?
> > 5) Faça uma boa questão sobre essa enciclopédia
> >
> > --
> > Esta mensagem foi verificada pelo sistema de antiv?rus e
> >  acredita-se estar livre de perigo.
> >
> > =
> > Instru??es para entrar na lista, sair da lista e usar a lista em
> > http://www.mat.puc-rio.br/~obmlistas/obm-l.html
> > =
>
> --
> Esta mensagem foi verificada pelo sistema de antivírus e
>  acredita-se estar livre de perigo.
>
> =
> Instru�ões para entrar na lista, sair da lista e usar a lista em
> http://www.mat.puc-rio.br/~obmlistas/obm-l.html
> =
>

-- 
Esta mensagem foi verificada pelo sistema de antiv�rus e
 acredita-se estar livre de perigo.



Re: [obm-l]

2019-11-12 Por tôpico Esdras Muniz
O caso "a" par eu fiz assim: a=2k, daí, (3^k)^2+ b^4=(d^2+1)^2, então vc
usa que para algum par p, q, com 0 escreveu:

> Será que não sai usando somente congruência módulo 8?
>
> Em ter., 12 de nov. de 2019 às 20:07, Pedro José 
> escreveu:
>
>> Boa noite!
>> Esdras,
>> tem como você postar, mesmo para o caso apenas de n par?
>>
>> Grato!
>>
>> Saudações,
>> PJMS.
>>
>> Em ter., 12 de nov. de 2019 às 19:52, Pedro José 
>> escreveu:
>>
>>> Boa noite!
>>> Carlos Gustavo,
>>> grato pela luz, estava tão obsecado e só rodando em círculos, tal qual
>>> patrulha perdida.
>>>
>>> Saudações,
>>> PJMS
>>>
>>> Em ter., 12 de nov. de 2019 às 19:19, Esdras Muniz <
>>> esdrasmunizm...@gmail.com> escreveu:
>>>
>>>> Dá para mostrar que a única solução com a e b pares é (2, 2). Agora com
>>>> a e b ímpares, não consegui.
>>>>
>>>> Em ter, 12 de nov de 2019 18:19, Pedro José 
>>>> escreveu:
>>>>
>>>>> Boa noite!
>>>>> Agora captei vosso pensamento.
>>>>> Só que ao transformar a equação em uma equação de Pell, nós maculamos
>>>>> a função 3^n.
>>>>> Em verdade a solução para a par a= 2n, seria (2,2); pois, como
>>>>> mencionara anteriormente se a é par, b também o é.
>>>>> Só que quando procuramos as outras soluções, baseando-se na
>>>>> propriedade de que a norma em Q [RAiz(A)] conserva a multiplicação. Só que
>>>>> quando eu pego a solução
>>>>> 3 + 2 Raiz(2) e elevo ao quadrado 17 + 12 Raiz(2). Se eu pegar
>>>>> 17^2-2*12^2=1 eu atendo x^2 - 2Y^2=1. E assim sucessivamente. Mas não
>>>>> existe n inteiro tal que 3^n=17, então não é uma solução da equação
>>>>> original.
>>>>> Creio que seja um pouco mais complicada a solução. Pois o difícil é
>>>>> saber quando atende também a 3^n.
>>>>> Acredito que deva haver uma forma de restringir a essas soluções,
>>>>> pois, definir em que condições a solução terá x como uma potência de 3 
>>>>> seja
>>>>> bem difícil.
>>>>> Estou apanhando mais do que mala velha em véspera de viagem.
>>>>> Se alguém postar uma solução, me ajudaria bastante.
>>>>>
>>>>> Saudações,
>>>>> PJMS
>>>>>
>>>>>
>>>>> Saudações,
>>>>> PJMS.
>>>>>
>>>>>
>>>>>
>>>>> Em ter., 12 de nov. de 2019 às 17:25, Pedro José 
>>>>> escreveu:
>>>>>
>>>>>> Boa tarde!
>>>>>> Douglas,
>>>>>> perdoe-me pela minha miopia, mas você poderia detalhar melhor onde
>>>>>> entra a equação de Pell?
>>>>>> A equação de Pell não é x^2-Dy^2 = N?
>>>>>> Se a é par b é par e se a ímpar b é ímpar para atender mod8,
>>>>>> Não consegui captar a sugestão.
>>>>>>
>>>>>> Saudações,
>>>>>> PJMS
>>>>>>
>>>>>> Em ter., 12 de nov. de 2019 às 16:50, Prof. Douglas Oliveira <
>>>>>> profdouglaso.del...@gmail.com> escreveu:
>>>>>>
>>>>>>> Hum, então, vamos analisar o caso de a ser par do tipo 2n.
>>>>>>>
>>>>>>> Assim podemos escrever que (3^n+b(sqrt2))(3^n-b(sqrt2))=1
>>>>>>> Dai através da solução mínima que o Pedro fez, como (1,1) por
>>>>>>> exemplo, da pra ver que são infinitas soluções usando a equação de Pell.
>>>>>>>
>>>>>>> Abraco
>>>>>>> Douglas Oliveira.
>>>>>>>
>>>>>>>
>>>>>>>
>>>>>>> Em dom, 10 de nov de 2019 19:33, gilberto azevedo <
>>>>>>> gil159...@gmail.com> escreveu:
>>>>>>>
>>>>>>>> [HELP]
>>>>>>>>
>>>>>>>> Achas todos os pares (a,b) inteiros positivos tais que :
>>>>>>>> 3^a = 2b² + 1.
>>>>>>>>
>>>>>>>>
>>>>>>>> --
>>>>>>>> Esta mensagem foi verificada pelo sistema de antivírus e
>>>>>>>> acredita-se estar livre de perigo.
>>>>>>>
>>>>>>>
>>>>>>> --
>>>>>>> Esta mensagem foi verificada pelo sistema de antivírus e
>>>>>>> acredita-se estar livre de perigo.
>>>>>>
>>>>>>
>>>>> --
>>>>> Esta mensagem foi verificada pelo sistema de antivírus e
>>>>> acredita-se estar livre de perigo.
>>>>
>>>>
>>>> --
>>>> Esta mensagem foi verificada pelo sistema de antivírus e
>>>> acredita-se estar livre de perigo.
>>>
>>>
>> --
>> Esta mensagem foi verificada pelo sistema de antivírus e
>> acredita-se estar livre de perigo.
>
>
> --
> Esta mensagem foi verificada pelo sistema de antivírus e
> acredita-se estar livre de perigo.

-- 
Esta mensagem foi verificada pelo sistema de antiv�rus e
 acredita-se estar livre de perigo.



Re: [obm-l]

2019-11-12 Por tôpico Esdras Muniz
Dá para mostrar que a única solução com a e b pares é (2, 2). Agora com a e
b ímpares, não consegui.

Em ter, 12 de nov de 2019 18:19, Pedro José  escreveu:

> Boa noite!
> Agora captei vosso pensamento.
> Só que ao transformar a equação em uma equação de Pell, nós maculamos a
> função 3^n.
> Em verdade a solução para a par a= 2n, seria (2,2); pois, como mencionara
> anteriormente se a é par, b também o é.
> Só que quando procuramos as outras soluções, baseando-se na propriedade de
> que a norma em Q [RAiz(A)] conserva a multiplicação. Só que quando eu pego
> a solução
> 3 + 2 Raiz(2) e elevo ao quadrado 17 + 12 Raiz(2). Se eu pegar
> 17^2-2*12^2=1 eu atendo x^2 - 2Y^2=1. E assim sucessivamente. Mas não
> existe n inteiro tal que 3^n=17, então não é uma solução da equação
> original.
> Creio que seja um pouco mais complicada a solução. Pois o difícil é saber
> quando atende também a 3^n.
> Acredito que deva haver uma forma de restringir a essas soluções, pois,
> definir em que condições a solução terá x como uma potência de 3 seja bem
> difícil.
> Estou apanhando mais do que mala velha em véspera de viagem.
> Se alguém postar uma solução, me ajudaria bastante.
>
> Saudações,
> PJMS
>
>
> Saudações,
> PJMS.
>
>
>
> Em ter., 12 de nov. de 2019 às 17:25, Pedro José 
> escreveu:
>
>> Boa tarde!
>> Douglas,
>> perdoe-me pela minha miopia, mas você poderia detalhar melhor onde entra
>> a equação de Pell?
>> A equação de Pell não é x^2-Dy^2 = N?
>> Se a é par b é par e se a ímpar b é ímpar para atender mod8,
>> Não consegui captar a sugestão.
>>
>> Saudações,
>> PJMS
>>
>> Em ter., 12 de nov. de 2019 às 16:50, Prof. Douglas Oliveira <
>> profdouglaso.del...@gmail.com> escreveu:
>>
>>> Hum, então, vamos analisar o caso de a ser par do tipo 2n.
>>>
>>> Assim podemos escrever que (3^n+b(sqrt2))(3^n-b(sqrt2))=1
>>> Dai através da solução mínima que o Pedro fez, como (1,1) por exemplo,
>>> da pra ver que são infinitas soluções usando a equação de Pell.
>>>
>>> Abraco
>>> Douglas Oliveira.
>>>
>>>
>>>
>>> Em dom, 10 de nov de 2019 19:33, gilberto azevedo 
>>> escreveu:
>>>
 [HELP]

 Achas todos os pares (a,b) inteiros positivos tais que :
 3^a = 2b² + 1.


 --
 Esta mensagem foi verificada pelo sistema de antivírus e
 acredita-se estar livre de perigo.
>>>
>>>
>>> --
>>> Esta mensagem foi verificada pelo sistema de antivírus e
>>> acredita-se estar livre de perigo.
>>
>>
> --
> Esta mensagem foi verificada pelo sistema de antivírus e
> acredita-se estar livre de perigo.

-- 
Esta mensagem foi verificada pelo sistema de antiv�rus e
 acredita-se estar livre de perigo.



Re: [obm-l]

2019-11-08 Por tôpico Esdras Muniz
Usa que f(x^2+x-3)=(x^3+2x^2-3x-5)f(x).

Em qui, 7 de nov de 2019 11:53, gilberto azevedo 
escreveu:

> [Polinômios]
>
> Dada a função f(x) = x³ + x² - 4x + 1 , mostrar que se f(r) = 0 , então
> f(r² + r -3) = 0.
> Creio que tem uma sacada pra aparecer r³ + r² - 4r + 1 , e usar que isso é
> 0, porém não estou enxergando.
>
> --
> Esta mensagem foi verificada pelo sistema de antivírus e
> acredita-se estar livre de perigo.

-- 
Esta mensagem foi verificada pelo sistema de antiv�rus e
 acredita-se estar livre de perigo.



[obm-l] Re: [obm-l] Re: [obm-l] Re: [obm-l] Minha solução para o item c) do problema 3 da prova da OBM-2017

2019-11-08 Por tôpico Esdras Muniz
Acho que é só passar 2017 para a base 6 e depois substituir os algarismos
0, 1, 2, 3, 4, 5 por 1, 3, 5, 7, 9 respectivamente.

Assim, 2017 na base 6 é 13201, trocando os algarismos, fica: 37513.

Em qui, 7 de nov de 2019 22:16, Cauã DSR  escreveu:

> Muito obrigado! É realmente uma honra ler isso.
> Sobre a questão eu ficarei de analisá-la (principalmente algumas funções
> que não entendi ainda) no sábado, se possível
>
> Em qui, 7 de nov de 2019 9:27 PM, Pedro José 
> escreveu:
>
>> Boa noite!
>>
>> Pode-se usar a soma da PG de razão 5 e o primeir termo 1
>>
>> então, no sistema impa, teremos 5 números com 1 algarismo, 30 números com
>> 1ou 2 algarismos, 155 números com até 3 algarismos, 780 números com até 4
>> algarismos e Sn=(5^n-1)/4 números com até n algarismos.
>>
>> Os algarismos de ordem mais baixa tem um padrão 1 3 5 7 9 1 3 5 7 9...
>> depois 1 3 5 7 9, depois ...1(5^2vezes)  3..33
>> .55
>> 77.777  e assim sucessivamente.
>>
>> Do algarismo menos significativo para o mais.
>> Até 5 só há um algarismo.
>> De 6= S2 em diante teremos pelo menos dois algarismos
>> De  31= S3 em diante teremos pelo menos dois algarismos.
>> De 156 = S4 em diante teremos pelo menos três algarismos
>>
>> De Sn+1 teremos pelo menos n algarismos.
>> Podemos achar os algarismos xn xn-1 ...x2 x1  para um número na base
>> decimal assim
>> se ai= 0 xi=1, se ai =1 xi=3; se ai=3 x1= 7 e se ai=4 xi=9 para i <=n
>> k1=Int (y-S1) e a1= mod (k1;5)
>> k2=int((y-S2)/5) e a2= mod(k2;5)
>>
>> kn=int((y-Sn)/ 5^(n-1)) e an = mod(kn;5)
>>
>> Para o número em questão: 2017
>> k1 = 2016 e a1=1  então x1=3
>> k2=int((2017-6)/5)=402; a2=2 então x2=5
>> k3=int((2017-31)/25)=79. a3=4 então x3=9
>> k4=int((2017-156)/125)=14; a4=4 e x4=9
>> k5=int((2017-781)/625)=1; a5=1 e x5=3
>>
>> Não há mais algarismos pois 2017 <3906=S6. Portanto a representação é:
>> 39953.
>>
>> Porém você, Cauã DSR ,deu uma ideia muito legal.
>>
>> Estou querendo provar duas coisas, que não consegui, mas estou certo que
>> acontece.
>>
>>
>> Se o número em decimal passado para base 5 não tiver algarismos zero,
>> você pode simplesmente.
>> 1 permanece 1 na impa
>> 2 vira 3 na impa
>> 3 vira 5 na impa
>> 4 vira 7 na impa.
>>
>> Caso você tenha um número com algarismo zero quando transformado para a
>> base 5,e.g., y= (x6x5x40x2x1)base5
>> Você pode,sendo o o indicado o menos significativo, Impa (y)=
>> Concat(impa(x6x5x40);impa(x2x1)) oNde concat é a concatenação.
>>
>> Assim para o nosso número original 2017= (31032) base5
>> impa (310)base 5
>> (310)base5=80
>> k1=79; a1=4 e x1=9
>> k2=int((80-6)/5=14 ;a2=4 e x2=9
>> k3=int(80-31)/25=1 a3= 1 e x3 = 3
>>
>> então impa (310)base5= 399
>> impa(32)= 53, faz direto pois não tem nenhum algarismo zero.
>> Então impa (31032)=Concat (impa(310);(impa(32))= 39953; como achado acima.
>>
>> É uma sacada legal. Pois; se não tem algarismo zero na base 5 sai direto.
>>
>> Caso haja você quebra o número o da direita sai direto. E na esquerda
>> você trabalha com um número menor.
>> Depois é só concatenar.
>> Só não consegui provar ainda. Sua ideia foi muito boa.
>>
>> Parabéns,
>> PJMS.
>>
>>
>>
>>
>>
>>
>>
>>
>>
>>
>>
>>
>> Em qui., 7 de nov. de 2019 às 17:27, Pedro José 
>> escreveu:
>>
>>> Boa tarde!
>>> Você seguiu uma linha de argumentação interessante.
>>> Mas não está correto.
>>> Pois existem 5 números com 1 algarismo 5^2 números com 2 algarismos, 5^3
>>> com 3 e assim sucessivamente.
>>> Usando a soma da PG
>>> 6-11
>>> 31 -111
>>> 156 -
>>> 781- 1
>>> Assim o maior número de 4 algarismos  representaria 780.
>>> O número teria que ter 5 algarismos.
>>>
>>> Saudações,
>>> PJMS
>>>
>>>
>>>
>>> Em qui., 7 de nov. de 2019 às 12:36, Cauã DSR 
>>> escreveu:
>>>

 Tenho um pequeno problema, eu fiz o item C) do problema 3 da prova da
 OBM de 2017, mas não tenho certeza sobre seu resultado, então achei uma boa
 fazer minha primeira aparição no grupo perguntando se o que fiz está certo.

 3. Na Terra dos Impas, somente os algarismos ímpares são utilizados
 para contar e escrever números. Assim, em vez dos numeros 1, 2, 3, 4, 5, 6,
 7, 8, 9, 10, 11, 12,. . . os Impas tem os números correspondentes 1, 3, 5,
 7, 9, 11, 13, 15, 17, 19, 31, 33, . . . (note que os números dos Impas tem
 somente algarismos ímpares). Por exemplo, se
 uma criança tem 11 anos, os Impas diriam que ela tem 31 anos.

 c) Escreva, na linguagem dos Impas, o numero que na nossa representação
 decimal é escrito como 2017.

 Minha solução:
 Como no problema só temos Ímpares para usar como algarismo {1,3,5,7,9},
 temos um sistema de numeração de base 5, porém com os algarismos ímpares ao
 invés da base 5 comumente usada {0,1,2,3,4}. Ao analisar isso decidi
 transformar 2017 em um número de Base 5 {1,2,3,4,5}, ao usar esta base,
 percebi que para transformar um número de Base Decimal em um de Base 5
 {1,2,3,4,5} é quase o mesmo 

Re: [obm-l] Bibliografia

2019-11-02 Por tôpico Esdras Muniz
Tem o "Elon fino" Análise real do Elon Lages Lima.
Sobre limite, tem muita biografia em outras línguas, tem um livro muito bom
em italiano, mas não sei se tem em inglês, o livro do Pagani.

Em sex, 1 de nov de 2019 17:01, Luiz Antonio Rodrigues <
rodrigue...@gmail.com> escreveu:

> Olá, pessoal!
> Tudo bem?
> Eu preciso de um favor: duas indicações bibliográficas:
>
> 1) Um livro de Cálculo que trate limites de uma maneira mais profunda.
>
> 2) Um livro de Análise Real para principiantes; eu tenho o do Rudin, mas
> acho que, no momento, ele é muito avançado para mim.
>
> Muito obrigado!
> Um abraço!
> Luiz
>
>
> --
> Esta mensagem foi verificada pelo sistema de antivírus e
> acredita-se estar livre de perigo.

-- 
Esta mensagem foi verificada pelo sistema de antiv�rus e
 acredita-se estar livre de perigo.



[obm-l] Re: [obm-l] Máximo e Mínimo de uma Função

2019-11-02 Por tôpico Esdras Muniz
O máximo e o mínimo dessa função dependem do domínio onde ela está
definida, por exemplo, se ela está definida em R-{0}, ela não tem máximo
nem mínimo. Isso interpretando que a questão quer literalmente o valor
máximo de f. Se interpretar que ela quer o valor de x para o qual f(x) é
máximo ou mínimo, dá problema tb, pois se por exemplo vc coloca o domínio
de f como o intervalo [n, infinito), nesse sentido máximo e mínimo vão ser
ao menos n, que é um número arbitrário. Pra mim, a questão não tem solução.

Em sáb, 2 de nov de 2019 13:53, Luiz Antonio Rodrigues <
rodrigue...@gmail.com> escreveu:

> Olá, pessoal!
> Bom dia!
> Estou tentando resolver o seguinte problema:
>
> É dada a função:
>
> f(x)=(1/x)+sen(x)
>
> Pergunta-se:
>
> Em quais intervalos abaixo é garantido que encontremos o máximo e o mínimo
> desta função?
>
> a) [-12;-3]
> b) (-2;-1)
> c) [-pi;pi]
> d) [pi;2pi]
> e) [5;+ infinito)
>
> Eu só consegui encontrar um ponto crítico em x=0.
> Ele não é o único, pois vi isso num gráfico da função.
> Não sei como resolver a equação f'(x)=0.
> Acho que estamos lidando com números complexos.
> Intervalos fechados fazem parte da solução?
> Pergunto isso porque foi minha resposta, que está errada.
> Estou confuso.
> Alguém pode me ajudar?
> Muito obrigado e um abraço!
> Luiz
>
>
>
> --
> Esta mensagem foi verificada pelo sistema de antivírus e
> acredita-se estar livre de perigo.

-- 
Esta mensagem foi verificada pelo sistema de antiv�rus e
 acredita-se estar livre de perigo.



[obm-l] Re: [obm-l] Séries e somatórios

2019-10-30 Por tôpico Esdras Muniz
O livro concrete mathematics fala disso.

Em qua, 30 de out de 2019 19:51, Alexandre Antunes <
prof.alexandreantu...@gmail.com> escreveu:

>
> Boa noite,
>
> Alguém tem alguma referência de livro/apostila sobre operações e
> propriedades "avançadas" sobre séries, somatórios, somatórios duplos, etc...
>
> Antecipadamente agradeço.
>
> Atenciosamente,
>
> Prof. Msc. Alexandre Antunes
> www alexandre antunes com br
>
> --
> Esta mensagem foi verificada pelo sistema de antivírus e
> acredita-se estar livre de perigo.

-- 
Esta mensagem foi verificada pelo sistema de antiv�rus e
 acredita-se estar livre de perigo.



[obm-l] Re: [obm-l] Triângulos.

2019-10-27 Por tôpico Esdras Muniz
Usa ma>=mg

Em dom, 27 de out de 2019 19:27, Guilherme Abbehusen <
gui.abbehuse...@gmail.com> escreveu:

> Olá, poderiam me ajudar com essa questão?
>
> A hipotenusa de um triângulo retângulo tem medida igual "a" e os catetos
> medidas iguais a "b" e "c" . Qual é o valor mínimo da equação: a/(b*c)^-1 ?
>
> Agradeco desde já.
>
> --
> Esta mensagem foi verificada pelo sistema de antivírus e
> acredita-se estar livre de perigo.

-- 
Esta mensagem foi verificada pelo sistema de antiv�rus e
 acredita-se estar livre de perigo.



Re: [obm-l] Polinomios

2019-10-27 Por tôpico Esdras Muniz
Dá pra provar por indicação, suponha q o resultado vale pra grau de P<=n-1.
Daí, use que entre um máximo e um mínimo de P, há no máximo uma raíz (é
fácil mostrar isso usando só a continuidade de P). Assim, por suposição, P
tem no máximo n+1 máximos, que são as raízes de P', + infinito e -
infinito.

Em sex, 25 de out de 2019 20:55, Israel Meireles Chrisostomo <
israelmchrisost...@gmail.com> escreveu:

>
> Alguém conhece um material ou mesmo a prova do teorema que diz todo
> polinômio de grau n não tem mais que n raízes reais?
> --
> Israel Meireles Chrisostomo
>
>
> 
>  Livre
> de vírus. www.avg.com
> .
> <#m_3807766465305337955_DAB4FAD8-2DD7-40BB-A1B8-4E2AA1F9FDF2>
>
> --
> Esta mensagem foi verificada pelo sistema de antivírus e
> acredita-se estar livre de perigo.

-- 
Esta mensagem foi verificada pelo sistema de antiv�rus e
 acredita-se estar livre de perigo.



Re: [obm-l] Cotangentes

2019-10-23 Por tôpico Esdras Muniz
Isso é falso, pois (2k-1)/4n forma uma seq crescente indo pro infinito,
então a cota gente ao quadrado forma uma seq decrescente indo pra zero.

Em qua, 23 de out de 2019 16:52, Israel Meireles Chrisostomo <
israelmchrisost...@gmail.com> escreveu:

> Como posso mostrar que cot²((2k-1)/4n) com k=1 até n, os termos dessa
> sequência se repetem a partir do  k=n+1
>
> --
> Israel Meireles Chrisostomo
>
>
> 
>  Livre
> de vírus. www.avg.com
> .
> <#m_7317635106403927221_DAB4FAD8-2DD7-40BB-A1B8-4E2AA1F9FDF2>
>
> --
> Esta mensagem foi verificada pelo sistema de antivírus e
> acredita-se estar livre de perigo.

-- 
Esta mensagem foi verificada pelo sistema de antiv�rus e
 acredita-se estar livre de perigo.



[obm-l] Re: [obm-l] Teorema Fundamental da álgebra prova

2019-10-10 Por tôpico Esdras Muniz
Se o polinômios tem grau ímpar, vc consegue mostrar que ele tem uma raíz
real, usando só a continuidade do polinômio. Tem tb uma demonstração
elementar de um caso particular do tfa, o caso em que n/4 e 3n/4 não são
quadrados perfeitos, onde n é o grau do polinômio.

Em qui, 10 de out de 2019 16:12, Israel Meireles Chrisostomo <
israelmchrisost...@gmail.com> escreveu:

>
> Existe alguma demonstração do Teorema Fundamental da Álgebra sem recorrer
> aos números complexos?Tipo uma versão mais fraca (para polinômio com
> coeficientes inteiros e raízes reais)?
> --
> Israel Meireles Chrisostomo
>
>
> 
>  Livre
> de vírus. www.avg.com
> .
> <#m_-2898584284853493076_DAB4FAD8-2DD7-40BB-A1B8-4E2AA1F9FDF2>
>
> --
> Esta mensagem foi verificada pelo sistema de antivírus e
> acredita-se estar livre de perigo.

-- 
Esta mensagem foi verificada pelo sistema de antiv�rus e
 acredita-se estar livre de perigo.



[obm-l] Re: [obm-l] Re: [obm-l] Congruência (?)

2019-10-04 Por tôpico Esdras Muniz
Dá pra fazer tb usando o pequeno teorema de Fermat.

<https://www.avast.com/sig-email?utm_medium=email_source=link_campaign=sig-email_content=webmail>
Livre
de vírus. www.avast.com
<https://www.avast.com/sig-email?utm_medium=email_source=link_campaign=sig-email_content=webmail>.
<#DAB4FAD8-2DD7-40BB-A1B8-4E2AA1F9FDF2>

Em sex, 4 de out de 2019 às 17:36, Pedro José 
escreveu:

> Boa tarde!
> Com minhas escusas retificação da solução.
> n<>o mod10 e não: "n<> 0 mod100"
> (100,4) <>1 e não: "(100,4) =1"
> b^x não se repete e não: "b^x não se repetem"
> Sds,
> PJMS.
>
>
> Em sex, 4 de out de 2019 às 17:16, Pedro José 
> escreveu:
>
>> Boa tarde!
>> Se 10 não divide n então n<>0 mod100; pois nesse caso daria "00".
>> Então os números são 2,4 ou 8 côngruos mod10.
>> 2^20=4^10
>> 8^20 = 4^40
>> 4^1= 4 mod10
>> 4^2=6 mod10
>> 4^3= 4 mod10
>> Logo temos que 4^(2m+1) = 4 mod 10 (i)
>> Se
>> a=4 mod 100 ==> a=4 mod 10 (ii)
>>
>> Então vamos procurar o período de a^n mod100,  Não existe a que satisfaça
>> a^m= 1 mod100, com m<>0, pois (100,4)=1
>> Vamos tentar verificar se há repetição do 4.
>> De (i) e (ii) , temos que: 4^(2m+1) = 4 mod 100
>> m=1 ==> 4^3 = 64 mod 104, não serve
>> m=2 ==> 4^5= (4^(3*2))*8 = 28*8= 224=24 mod 100, não serve
>> m=3 ==> 4^7= 24*16=384=84 mod 100
>> m=4 ==> 4^9= (2*84)*8=68*8= 544=44 mod100
>> m=5 ==> 4^11=44*16= 704= 4 mod 100
>> Portanto o período de 4^a mod100 é 1gual a 10, ou seja, 4^a=4(10x+a)
>> mod100. com x,a não nulos (Cuidado, que para alguns casos em que (b,m)<>1,
>> b^x não se repetem para x < xo,e.g., 2^a= 2 mod 100, só é atendido para
>> a=1, aí tem de sair no braço para ver qual que se repete e pode-se gastar
>> mais tempo. Por sorte o quatro repetiu. Mas o enunciado dava a dica de que
>> repetiria, pois, 4^20=4^10 mod 100 para que o problema tenha uma solução
>> única.
>> 4^20 = 4^10= 4^9*4=44*4=176=76 mod100
>> 8^20=4^40=4^10=76 mod100
>> 2^20=4^10=76 mod 100.
>>
>> Portanto o algarismo da dezena é 7 e das unidades 6.
>>
>> Saudações,
>> PJMS
>>
>>
>>
>>
>> Em qui, 3 de out de 2019 às 17:51, marcone augusto araújo borges <
>> marconeborge...@hotmail.com> escreveu:
>>
>>> Se n é um número natural par não divisível por 10, quais são os dois
>>> últimos algarismos de n^20?
>>> --
>>> Esta mensagem foi verificada pelo sistema de antivírus e
>>> acredita-se estar livre de perigo.
>>>
>>
> --
> Esta mensagem foi verificada pelo sistema de antivírus e
> acredita-se estar livre de perigo.



-- 
Esdras Muniz Mota
Mestrando em Matemática
Universidade Federal do Ceará

-- 
Esta mensagem foi verificada pelo sistema de antiv�rus e
 acredita-se estar livre de perigo.



[obm-l] Re: [obm-l] Questão sobre equações funcionais

2019-07-28 Por tôpico Esdras Muniz
Errei, satisfaz sim :)

Em dom, 28 de jul de 2019 14:21, Esdras Muniz 
escreveu:

> Mas essa função que VC achou não satisfaz a igualdade.
>
> Em dom, 28 de jul de 2019 01:05, Carlos Monteiro <
> cacacarlosalberto1...@gmail.com> escreveu:
>
>> (Questão) Encontre todas as funções f : R-> R tais que
>>  f(xy - f(x)) = x.f(y)
>>
>> Minha tentativa, não sei se está correta:
>> I) p(x, f(x)/(x-1)): f( f(x)/(x-1) ) = 0; x diferente de 1
>>
>> II) Seja c um número real tal que f(c)=0
>>   i) fazendo x=c na equação encontrada em I: *c diferente de 1*
>> f(0)=0
>>
>>  Se f(0)=0
>> p(x, f(x)/x): 0=x.f( f(x)/x ); x diferente de 0 .: f(x)/x=0
>>  f(x)=0, para todo x real.
>>
>>ii) p(c,1) na equação dada no problema:
>>   0=c.f(1) <=> c=0(caso anterior) ou f(1)=0
>>
>>   Se f(1)=0, 1 é raíz única
>>f( f(x)/(x-1) )=0 .: f(x)/(x-1) = 1 => f(x)=x-1, para todo x real.
>>
>>
>>
>>
>> --
>> Esta mensagem foi verificada pelo sistema de antivírus e
>> acredita-se estar livre de perigo.
>
>

-- 
Esta mensagem foi verificada pelo sistema de antiv�rus e
 acredita-se estar livre de perigo.



[obm-l] Re: [obm-l] Questão sobre equações funcionais

2019-07-28 Por tôpico Esdras Muniz
Mas essa função que VC achou não satisfaz a igualdade.

Em dom, 28 de jul de 2019 01:05, Carlos Monteiro <
cacacarlosalberto1...@gmail.com> escreveu:

> (Questão) Encontre todas as funções f : R-> R tais que
>  f(xy - f(x)) = x.f(y)
>
> Minha tentativa, não sei se está correta:
> I) p(x, f(x)/(x-1)): f( f(x)/(x-1) ) = 0; x diferente de 1
>
> II) Seja c um número real tal que f(c)=0
>   i) fazendo x=c na equação encontrada em I: *c diferente de 1*
> f(0)=0
>
>  Se f(0)=0
> p(x, f(x)/x): 0=x.f( f(x)/x ); x diferente de 0 .: f(x)/x=0
>  f(x)=0, para todo x real.
>
>ii) p(c,1) na equação dada no problema:
>   0=c.f(1) <=> c=0(caso anterior) ou f(1)=0
>
>   Se f(1)=0, 1 é raíz única
>f( f(x)/(x-1) )=0 .: f(x)/(x-1) = 1 => f(x)=x-1, para todo x real.
>
>
>
>
> --
> Esta mensagem foi verificada pelo sistema de antivírus e
> acredita-se estar livre de perigo.

-- 
Esta mensagem foi verificada pelo sistema de antiv�rus e
 acredita-se estar livre de perigo.



Re: [obm-l] desigualdades

2019-06-12 Por tôpico Esdras Muniz
Provar que E=a/(a+b)  +  b/(b+c) +  c/(c+d)   +   d/(d+a)  <  3

Se u, v e k são positivos, com uhttps://www.avast.com/sig-email?utm_medium=email_source=link_campaign=sig-email_content=webmail>
Livre
de vírus. www.avast.com
.
<#DAB4FAD8-2DD7-40BB-A1B8-4E2AA1F9FDF2>


Livre
de vírus. www.avast.com
.
<#m_-2965441995128554056_DAB4FAD8-2DD7-40BB-A1B8-4E2AA1F9FDF2>

-- 
Esta mensagem foi verificada pelo sistema de antiv�rus e
 acredita-se estar livre de perigo.



[obm-l] Re: [obm-l] Torneio das Cidades ( Número mínimo de Tentativas )

2019-02-24 Por tôpico Esdras Muniz
Já vi um problema parecido da OBM que dava pra resolver usando o teorema de
Turan. Pra esse imaginei assim, VC separa as baterias em dois grupos, B das
boas e R das ruins e liga duas se foram testadas juntas. O numero máximo de
arestas que dá pra colocar sem a lampada acender é |B|×|R| +|R|×(|R|-1)/2.

Em dom, 24 de fev de 2019 11:40, Jeferson Almir 
escreveu:

> Peço ajuda aos amigos da lista, sei que existe um problemas da obm
> "parecido",  aguardo dicas ou soluções. Eu tentei formar um grafo de
> tentativas e penso como otimizar ele.
>
> a.) Existem 2n + 1 (n> 2) baterias. Não sabemos quais baterias são boas e
> quais são ruins, mas sabemos que o número de baterias boas é maior do que o
> número de baterias ruins. Uma lâmpada usa duas baterias e só funciona se
> ambas forem boas. Qual é o menor número de tentativas suficientes para
> fazer a lâmpada funcionar?
>
> b.) O mesmo problema, mas o número total de baterias é 2n (n> 2) e os
> números de baterias boas e ruins são iguais.
>
>
>
> --
> Esta mensagem foi verificada pelo sistema de antivírus e
> acredita-se estar livre de perigo.

-- 
Esta mensagem foi verificada pelo sistema de antiv�rus e
 acredita-se estar livre de perigo.



[obm-l] Re: [obm-l] Re: [obm-l] Recorrência de 2ª Ordem

2019-02-15 Por tôpico Esdras Muniz
Suponha que a equação seja Xn+2=2aXn+1-a^2Xn, então,
(Xn+2-aXn+1)=a(Xn+1-aXn). Defina Yn=Xn+1-aXn. Daí, Yn+1=aYn, então fica
Yn=B.a^n. Xn+1=aXn+B.a^n. Que é uma equação de primeira ordem.

Em sex, 15 de fev de 2019 00:11, Claudio Buffara  Pelo método experimental.
>
> Suponhamos que você já conheça a fórmula do termo geral quando as raízes
> são simples.
>
> Suponhamos também que a recorrência que você quer resolver tenha uma
> equação característica com uma raiz dupla k.
>
> Com uma planilha, calcule x(n) para 1 <= n <= 20 (ou algo assim).
> Daí, calcule y(n) = x(n)/k^n e observe que y(n) é uma PA.
>
> []s,
> Claudio.
>
>
>
> On Thu, Feb 14, 2019 at 11:41 PM Jeferson Almir 
> wrote:
>
>> Olá companheiros da lista, quando nos deparamos com uma recorrência de
>> segunda ordem e na tentativa de resolvê-la montamos um equação ou polinômio
>> característico, e minha dúvida está em  saber como deduzir a solução da
>> equação de recorrência para o caso em que as raizes são iguaispois o
>> caso em que ela são diferentes eu consegui, se vc jogar a solução x = (A +
>> Bn)(lambda)^n dá perfeito  mas como chegar nela ?
>>
>> --
>> Esta mensagem foi verificada pelo sistema de antivírus e
>> acredita-se estar livre de perigo.
>
>
> --
> Esta mensagem foi verificada pelo sistema de antivírus e
> acredita-se estar livre de perigo.

-- 
Esta mensagem foi verificada pelo sistema de antiv�rus e
 acredita-se estar livre de perigo.



[obm-l] Re: [obm-l] trigonometria, alguma sugestão?

2018-09-28 Por tôpico Esdras Muniz
Usando a notação cis(u)=cos(u)+i.sen(u), temos que cis(u).cis(v)=cis(u+v),
para todos os u, v reais.
Daí, 1 = b/a.a/c.c/b = cis(x).cis(y)cis(z) = cis(x+y+z). Então cos(x+y+z) =
1 e sen(x+y+z) = 0. Portanto (x+y+z) é múltiplo de 2π.

Em qui, 27 de set de 2018 às 22:11, Israel Meireles Chrisostomo <
israelmchrisost...@gmail.com> escreveu:

>   Prove que se existem números complexos a,b e c tais que  b/a = cos(x) +
> isen(x),  a/c = cos(y) + isen(y)  e  c/b = cos(z) + isen(z)
>
>   Então existe um valor de j pertencente aos naturais, tal que para cada
> valor de k natural a igualdade x + y + z = 2jπ/k é verdadeira.
>
> --
> Israel Meireles Chrisostomo
>
> --
> Esta mensagem foi verificada pelo sistema de antivírus e
> acredita-se estar livre de perigo.



-- 
Esdras Muniz Mota
Mestrando em Matemática
Universidade Federal do Ceará

-- 
Esta mensagem foi verificada pelo sistema de antiv�rus e
 acredita-se estar livre de perigo.



[obm-l] Re: [obm-l] Polinômios ( RPM)

2018-09-20 Por tôpico Esdras Muniz
Suponha por absurdo que (7-Ri)>=0 para toda raiz Ri, i=1,...,100.
Daí, por Ma>=Mg, temos:
1>=\sqer[100]{(7-R1)(7-R2)...(7-R100)}>1 então 1>1, o que é um absurdo.

Em sex, 21 de set de 2018 às 01:05, Jeferson Almir 
escreveu:

> Este problema é de uma R.P.M que não sei qual o exemplar e peço ajuda.
>
> Seja P(x) um polinômio de grau 100 tal que P(x) = x^100 -600x^99 +
> 98x^98+97x^97 +... + a_1x + a_o tem 100 raizes reais e que P(7) > 1 .
> Mostre que existe pelo menos uma raiz maior que 7 .
>
> Desconfio muito de usar médias mas não estou conseguindo adequar para
> aplica-las .
>
> --
> Esta mensagem foi verificada pelo sistema de antivírus e
> acredita-se estar livre de perigo.



-- 
Esdras Muniz Mota
Mestrando em Matemática
Universidade Federal do Ceará

-- 
Esta mensagem foi verificada pelo sistema de antiv�rus e
 acredita-se estar livre de perigo.



Re: [obm-l] geometria plana

2018-07-29 Por tôpico Esdras Muniz
Pra mim não é tão fácil ver 3, to enferrujado na geometria plana. Pra
justificar acho que uma boa forma de ver é dizer que o triângulo APQ é
congruente ao PBC. Para concluir, o caminho mais curto que eu vi foi usar
que o triângulo MNR é semelhante ao BAN, e a razão é 1/2.

Em 28 de julho de 2018 20:51, Claudio Buffara 
escreveu:

> Idéia que me ocorreu: todo triângulo é afim-equivalente a um triângulo
> equilátero.
> Mediante translações, as medianas de um triângulo equilátero de lado 1
> formam um triângulo equilátero cujos lados medem raiz(3)/2 e, portanto,
> cuja área é 3/4.
> Será que uma transformação afim preserva a razão entre as áreas do
> triângulo original e do triângulo “medianico”?
>
> Abs,
> Cláudio.
>
> Enviado do meu iPhone
>
> Em 28 de jul de 2018, à(s) 19:08, matematica10complicada <
> profdouglaso.del...@gmail.com> escreveu:
>
> Então,podemos fazer o seguinte:
>
> Considere um triângulo ABC, cujas medianas são AM, BN, CP, e baricentro
> G desta forma
>
> 1)Monte um paralelogramo BNQM de forma que MQ intercepte AC em R.
>
> 2)Como o baricentro divide em seis áreas iguais, temos que a área do
> triângulo AGN será 1/6.
>
> 3)É fácil ver que MQ=BN, e AQ=CP.
>
> 4) Desta forma a área procurada será a do triângulo AMQ que é o dobro
> da área do triângulo AMR=3/8.
>
> Portanto a resposta é 3/4.
>
>
> Douglas Oliveira.
> Grande Abraço.
>
> Em 28 de julho de 2018 16:32, marcone augusto araújo borges <
> marconeborge...@hotmail.com> escreveu:
>
>> Seja um triangulo ABC cuja area eh igual a 1. Determinar a area do
>> triangulo cujos  lados sao iguais às medianas do triangulo ABC
>>
>> --
>> Esta mensagem foi verificada pelo sistema de antivírus e
>> acredita-se estar livre de perigo.
>>
>
>
> --
> Esta mensagem foi verificada pelo sistema de antivírus e
> acredita-se estar livre de perigo.
>
>
> --
> Esta mensagem foi verificada pelo sistema de antivírus e
> acredita-se estar livre de perigo.
>



-- 
Esdras Muniz Mota
Mestrando em Matemática
Universidade Federal do Ceará

-- 
Esta mensagem foi verificada pelo sistema de antiv�rus e
 acredita-se estar livre de perigo.



[obm-l] Re: [obm-l] área de triângulo( compartilhando)

2018-05-13 Por tôpico Esdras Muniz
Se u é o ângulo entre os lados de comprimento a e b, temos:
S = a*b*sen(u)/2 = (a^2+b^2)/4.
Daí, pela condição de igualdade entre as médias geométrica e aritmética,
temos que
sen(u)=1 e a=b. Logo os ângulos do triângulo são 90°, 45°, 45°.

Em 13 de maio de 2018 23:52, marcone augusto araújo borges <
marconeborge...@hotmail.com> escreveu:

> As medidas de dois lados de um triângulo são a e b e sua área é igual a
> (a^2+b^2)/4
>
> Determine os ângulos do triângulo
>
> --
> Esta mensagem foi verificada pelo sistema de antivírus e
> acredita-se estar livre de perigo.
>



-- 
Esdras Muniz Mota
Mestrando em Matemática
Universidade Federal do Ceará

-- 
Esta mensagem foi verificada pelo sistema de antiv�rus e
 acredita-se estar livre de perigo.



[obm-l] Re: [obm-l] Re: [obm-l] Re: [obm-l] Fwd: não sei como fazer, tentei desigualdades de médias e não saiu

2018-05-13 Por tôpico Esdras Muniz
Existem 85 triplas (p, q, r) com p<q escreveu:

>
> Em dom, 13 de mai de 2018 às 20:12, Pacini Bores <pacini.bo...@globo.com>
> escreveu:
>
>> Desculpe-me esqueci d colocar um dado na questão na hora d escrever. Os
> números p, q é r são primos ímpares. Havia colocado apenas ímpares.
>
>> Oi Daniel,
>>
>> Estranho, pois p=999, q= 1001 e r =1; teremos  p+q+r=2001 , pqr+1=
>> 100= (1000)^2.
>>
>> Ou seja, k=1000 ?
>>
>> Pacini
>>
>> Em 13/05/2018 2:56, Daniel Quevedo escreveu:
>>
>>
>>
>> - Mensagem encaminhada -
>> De: Daniel Quevedo <daniel...@gmail.com>
>> Data: dom, 13 de mai de 2018 às 02:54
>> Assunto:
>> Para: ob...@mat-puc.rio.br <ob...@mat-puc.rio.br>
>>
>>
>> Sabendo que p, q e r são números impares distintos com p+q+r= 2001 e que
>> k é um inteiro positivo tal que pqr +1 =k^2, a soma dos algarismos do único
>> valor possível para k é igual a:
>> A) 20
>> B) 21
>> C) 22
>> D) 23
>> E) 24
>> --
>> Fiscal: Daniel Quevedo
>> --
>> Fiscal: Daniel Quevedo
>>
>> --
>> Esta mensagem foi verificada pelo sistema de antivrus e
>> acredita-se estar livre de perigo.
>>
>>
>>
>> --
>> Esta mensagem foi verificada pelo sistema de antivírus e
>> acredita-se estar livre de perigo.
>>
> --
> Fiscal: Daniel Quevedo
>
> --
> Esta mensagem foi verificada pelo sistema de antivírus e
> acredita-se estar livre de perigo.
>



-- 
Esdras Muniz Mota
Mestrando em Matemática
Universidade Federal do Ceará

-- 
Esta mensagem foi verificada pelo sistema de antiv�rus e
 acredita-se estar livre de perigo.



Re: [obm-l] Combinatoria - quantas sequencias de comprimento "n" , com "p" elementos

2018-02-27 Por tôpico Esdras Muniz
p^n-(p-1)^n


Em 27 de fev de 2018 09:57, "Claudio Buffara" 
escreveu:

> Isso é igual ao número de sobrejeções de um conjunto com n elementos num
> conjunto com p elementos.
>
> É igual a p!*S(n,p), onde S(n,p) é o número de Stirling do 2o tipo (=
> número de partições de um conjunto com n elementos em p subconjuntos não
> vazios)
>
> Veja aqui: http://nptel.ac.in/courses/04026/lecture9.pdf  (eles usam
> m ao invés de p).
>
> []s,
> Claudio.
>
>
> 2018-02-27 7:25 GMT-03:00 Rogerio Ponce da Silva :
>
>> Ola' pessoal !
>> Existem quantas sequencias (diferentes entre si) de comprimento "n" ,
>> empregando-se somente "p" elementos, pelo menos uma vez cada um deles?
>>
>> []'s
>> Rogerio Ponce
>>
>> --
>> Esta mensagem foi verificada pelo sistema de antivírus e
>>  acredita-se estar livre de perigo.
>>
>> =
>> Instruções para entrar na lista, sair da lista e usar a lista em
>> http://www.mat.puc-rio.br/~obmlistas/obm-l.html
>> =
>>
>
>
> --
> Esta mensagem foi verificada pelo sistema de antivírus e
> acredita-se estar livre de perigo.

-- 
Esta mensagem foi verificada pelo sistema de antiv�rus e
 acredita-se estar livre de perigo.



[obm-l] Re: [obm-l] Dúvida combinatória

2017-11-14 Por tôpico Esdras Muniz
Obs:

$$S(n,\,k_1,\cdots ,k_n)=\frac{n!}{(k_1!\cdots k_n!)(1!)^{k_1}\cdots
(n!)^{k_n}}$$

-- 
Esta mensagem foi verificada pelo sistema de antiv�rus e
 acredita-se estar livre de perigo.



[obm-l] Re: [obm-l] Dúvida combinatória

2017-11-14 Por tôpico Esdras Muniz
Bem, imagine que vc tem [image: n] bolas iguais e quer distribuí-las em
caixas de tamanhos [image: k_1,\,k_2,\,\cdots,k_n], onde na caixa [image:
k_i] cabe [image: i] bolas, e você quer que no final cada caixa esteja
totalmente cheia ou vazia. Isso é equivalente ao problema que você propõe,
e a resposta é:

[image: S(n,\,k_1,k_2,\cdots,k_t)=\frac{n!}{(k_1!\cdots
k_t!)(1!)^{k_1}\cdots(n!)^{k_n}}]

.

Em 13 de novembro de 2017 23:30, Eduardo Henrique <dr.dhe...@outlook.com>
escreveu:

> Pessoal, estava estudando o seguinte tipo de problema:
>
> Quantas são as soluções inteiras positivas de a+b+c=r, com r inteiro
> positivo. Até aqui ok. A dúvida veio depois:
>
> Quantas são as solução inteiras positivas de 1a+2b+3c=r? E mais
> geralmente, de 1k_1+...+n_kn=r? Alguém sabe como abordar esse tipo de
> problema ou então saberia me indicar um material de estudos?
>
> Obrigado.
>
> Eduardo
>
> --
> Esta mensagem foi verificada pelo sistema de antivírus e
> acredita-se estar livre de perigo.
>



-- 
Esdras Muniz Mota
Mestrando em Matemática
Universidade Federal do Ceará

-- 
Esta mensagem foi verificada pelo sistema de antiv�rus e
 acredita-se estar livre de perigo.



Re: [obm-l] Problema de grafos

2017-09-02 Por tôpico Esdras Muniz
Cada vértice pode ter como grau um número de 0 a n-1, porém o 0 e o n-1 não
podem ambos ser graus de vértices, pois se um tem grau n-1 então ele está
ligado a todos os outros vértices. Então há apenas n-1 possibilidades para
o grau de cada vértice. Pelo pcp há dois vértices com o mesmo grau.

Em 2 de set de 2017 12:34 PM, "Daniel Rocha" 
escreveu:

> Bom dia,
>
> Seja G um grafo com n vértices, n maior que 1. Suponha que G não possua
> loops nem mais de uma aresta unindo pares de vértices. Prove que G possui
> dois vértices de graus iguais.
>
> Obrigado,
> Daniel
> --
> Esta mensagem foi verificada pelo sistema de antivírus e
>  acredita-se estar livre de perigo.
>
>
> =
> Instruções para entrar na lista, sair da lista e usar a lista em
> http://www.mat.puc-rio.br/~obmlistas/obm-l.html
> =
>

-- 
Esta mensagem foi verificada pelo sistema de antiv�rus e
 acredita-se estar livre de perigo.



Re: [obm-l] Fibonacci teoria dos numeros

2017-08-31 Por tôpico Esdras Muniz
Usa que f_{(m,n)}=(f_m, f_n)
Onde (a,b)=mdc(a,b).

Em 31 de agosto de 2017 16:30, Douglas Oliveira de Lima <
profdouglaso.del...@gmail.com> escreveu:

> Olá, como posso mostrar que para algum inteiro e positivo n, existe um
> número de Fibonacci que é múltiplo de n?
>
> Douglas Oliveira.
>
> --
> Esta mensagem foi verificada pelo sistema de antivírus e
> acredita-se estar livre de perigo.




-- 
Esdras Muniz Mota
Mestrando em Matemática
Universidade Federal do Ceará

-- 
Esta mensagem foi verificada pelo sistema de antiv�rus e
 acredita-se estar livre de perigo.



Re: [obm-l] a quem possa interessar

2017-08-10 Por tôpico Esdras Muniz
Caramba, é muita coisa! Obrigado.

Em 10 de agosto de 2017 09:27, Carlos Gomes <cgomes...@gmail.com> escreveu:

> Obrigado por compartilhar Murício...uma mina de ouro!
>
> abraço, Cgomes.
>
> Em 9 de agosto de 2017 22:36, Mauricio de Araujo <
> mauricio.de.ara...@gmail.com> escreveu:
>
>> https://drive.google.com/drive/folders/0B8qeUE5SqcPAWFVaM1N5anN3S2M
>>
>>
>>
>> --
>> Esta mensagem foi verificada pelo sistema de antivírus e
>> acredita-se estar livre de perigo.
>
>
>
> --
> Esta mensagem foi verificada pelo sistema de antivírus e
> acredita-se estar livre de perigo.
>



-- 
Esdras Muniz Mota
Mestrando em Matemática
Universidade Federal do Ceará

-- 
Esta mensagem foi verificada pelo sistema de antiv�rus e
 acredita-se estar livre de perigo.



[obm-l] Re: [obm-l] Re: [obm-l] Re: [obm-l] Sugestão de material para OBM

2017-07-04 Por tôpico Esdras Muniz
Cara, vc deve começar pegando um bom livro de cálculo, recomendo o Calculus
do Apostol, nele vc vai aprender também álgebra linear e geometria
analítica. São dois volumes, é material para passar um ano estudando.
Enquanto isso vc pode também ir estudando pelo livro da Putnam, que o
Carlos Gomes citou, ele vai te dar uma visão geral do que é preciso saber
pra enfrenar as provas da olimpíada universitária. Não desanime por não
estar em um curso de exatas, Fermat é um dos matemáticos mais famosos da
história, e era um matemático amador...

Em 4 de julho de 2017 15:48, Pedro José <petroc...@gmail.com> escreveu:

> Boa tarde!
>
> Desculpe-me, pela intromissão. Mas você, que não é da área de exatas, fica
> difícil enveredar de cara no nível universitário.
> Procure começar pelo nível médio. Os problemas já são cascas-grossas.
> Primeiro se erguer, depois andar e por fim correr, é o que costumo dizer a
> minha filha.
>
> Saudações,
> PJMS
>
>
> Em 4 de julho de 2017 12:56, Carlos Gomes <cgomes...@gmail.com> escreveu:
>
>> Olá Max...não seu o seu histórico anterior com as Olimpiadas...mas nunca
>> é tarde para começar...para o nivel universitário uma ótima referência é o
>> livro (PUTNAM BEYOND)  https://libgen.pw/download.php?id=10688 . Uma
>> outra boa dica é o proprio site da OBM onde você encontra a revista EUREKA
>> e também as provas anteriores.
>>
>> Um grande abraço, Cgomes.
>>
>> Em 4 de julho de 2017 10:23, Max Alexandre <maxmalexan...@gmail.com>
>> escreveu:
>>
>>> Olá, pesoal!
>>>
>>> Sou iniciante na área de olimpíadas. Faço faculdade na área da saúde,
>>> então não verei muito calculo ao longo do curso. Mas estou realmente
>>> instigado a resolver problemas de matemática. Já até me inscrevi na OBM e
>>> na OMERJ. Por isso, peço encarecidamente sugestões de materias voltados a
>>> olimpíadas de matemática de nível universitário visando uma preparação mais
>>> sólida pras competições.
>>>
>>> Desde já, agradeço.
>>>
>>> --
>>> Esta mensagem foi verificada pelo sistema de antivírus e
>>> acredita-se estar livre de perigo.
>>
>>
>>
>> --
>> Esta mensagem foi verificada pelo sistema de antivírus e
>> acredita-se estar livre de perigo.
>>
>
>
> --
> Esta mensagem foi verificada pelo sistema de antivírus e
> acredita-se estar livre de perigo.
>



-- 
Esdras Muniz Mota
Mestrando em Matemática
Universidade Federal do Ceará

-- 
Esta mensagem foi verificada pelo sistema de antiv�rus e
 acredita-se estar livre de perigo.



Re: [obm-l] Radicais

2017-06-04 Por tôpico Esdras Muniz
Comece com a identidade: $n^2\,=\,1+(n-1)\sqrt{(n+1)^2}$.

$3=\sqrt{3^2}=\sqrt{1+2\sqrt{4^2}}=\sqrt{1+2\sqrt{1+3\sqrt{5^2}}}=\cdots$

Em 4 de junho de 2017 15:40, Douglas Oliveira de Lima <
profdouglaso.del...@gmail.com> escreveu:

> Opa amigo, o radical do Indiano Ramanujam, baixe um arquivo do Carlos
> Victor , muito bom tem esse problema resolvido e vários outros.
> Segue o link http://cursos.ufrrj.br/posgraduacao/profmat/
> dissertacoes/dissertacoe/
>
> Um abraço
> Douglas Oliveira.
>
> Em 4 de jun de 2017 3:19 PM, "Pedro Júnior" <pedromatematic...@gmail.com>
> escreveu:
>
>> Olá pessoal, vocês poderiam me ajudar a solucionar o problema abaixo? Já
>> vi alguns bem parecidos, mas esse está me pegando...
>>
>> Raiz (1+2Raiz(1+3Raiz(1+4Raiz(1+...= ?
>>
>> Desde já agradeço
>>
>> --
>> Esta mensagem foi verificada pelo sistema de antivírus e
>> acredita-se estar livre de perigo.
>
>
> --
> Esta mensagem foi verificada pelo sistema de antivírus e
> acredita-se estar livre de perigo.
>



-- 
Esdras Muniz Mota
Mestrando em Matemática
Universidade Federal do Ceará

-- 
Esta mensagem foi verificada pelo sistema de antiv�rus e
 acredita-se estar livre de perigo.



Re: [obm-l] Desigualdade

2017-05-28 Por tôpico Esdras Muniz
Se vc colocar a1 igual a 0, 1 ou 2 vai ver queisso não é verdade. Acho que
é verdade se |a1|>e.

Em 28 de mai de 2017 11:58, "Douglas Oliveira de Lima" <
profdouglaso.del...@gmail.com> escreveu:

Então amigos, eu tive uma idéia mas não estou conseguindo concluir, vamos
lá:

Montei uma sequência e fiz a_1=3, e assim
[2^(1/2)].[3^(1/4)].[4^(1/8)].[5^(1/16)]...<3
se, e somente se,
[3^(1/4)].[4^(1/8)].[5^(1/16)]...<(3^2)/2, portanto a_2=(3^2)/2, e assim
sucessivamente escrevi a sequência
a_n=(a_(n-1)^2)/n, e usei um "leminha" para resolver, que é (a_n)^2>n+1, e
assim sai fácil, só não consegui escrever
a prova desse lema.

Mas com ele sai bem facil, pois se  (a_n)^2>n+1, então (a_n)>(n+1)^(1/2),
logo  (a_(n-1)^2)/n>(n+1)^(1/2), ou seja,
a_(n-1)>n^(1/2)(n+1)^(1/4), .,
a_1>[2^(1/2)].[3^(1/4)].[4^(1/8)].[5^(1/16)]...,
e como a_1=3, está provado.

Peço a ajuda de vocÊs para provar o lema.

 Lema:
Considere a sequência a_n=(a_(n-1)^2)/n, onde a_1=3 então (a_n)^2>n+1.



Douglas Oliveira




Em 27 de maio de 2017 18:10, Esdras Muniz <esdrasmunizm...@gmail.com>
escreveu:

> Solução muito boa.
>
> Em 27 de mai de 2017 00:37, "Gabriel Tostes" <gtos...@icloud.com>
> escreveu:
>
>> Tira ln, esse produto vai ser:
>> Sum{n>=1} ln(n+1)/(2^n) = M
>>
>> Bora escrever M de outro jeito:
>>
>> M= ln(2) + [ln(3)-ln(2)]/2 + [ln(4)-ln(3)]/2^2 + ...
>>
>> M= Sum{n>=1} (ln(n+1)-ln(n))/2^(n-1)
>>
>> Como ln(n+1)-ln(n)=ln(1+1/n)<1/n
>>
>> M=2} 1/n.2^(n-1) = L + ln(2)
>>
>> Para achar L considere:
>> 1/(1-x)= 1+x^2+x^3+...
>>
>> Integrando essa expressao temos que -(1/x).ln(1-x)= 1+x/2+x^2/3+...
>> Substituindo x=1/2 achamos que L=2ln(2)-1
>> E entao
>> M< 3ln(2)-1 < ln(3)
>>
>>  E o produto pedido inicialmente eh menor que 3
>>
>>
>>
>>
>>
>>
>>
>>
>> Sent from my iPad
>> > On May 26, 2017, at 9:47 PM, Douglas Oliveira de Lima <
>> profdouglaso.del...@gmail.com> wrote:
>> >
>> > Como posso fazer essa daqui:
>> >
>> > [2^(1/2)].[3^(1/4)].[4^(1/8)].[5^(1/16)]...<3
>> >
>> > Grande abraço a todos
>> >
>> > DouglasOliveira
>> >
>> > --
>> > Esta mensagem foi verificada pelo sistema de antivírus e
>> > acredita-se estar livre de perigo.
>>
>> --
>> Esta mensagem foi verificada pelo sistema de antivírus e
>>  acredita-se estar livre de perigo.
>>
>>
>> =
>> Instruções para entrar na lista, sair da lista e usar a lista em
>> http://www.mat.puc-rio.br/~obmlistas/obm-l.html
>> =
>>
>
> --
> Esta mensagem foi verificada pelo sistema de antivírus e
> acredita-se estar livre de perigo.
>


-- 
Esta mensagem foi verificada pelo sistema de antivírus e
acredita-se estar livre de perigo.

-- 
Esta mensagem foi verificada pelo sistema de antiv�rus e
 acredita-se estar livre de perigo.



Re: [obm-l] Desigualdade

2017-05-27 Por tôpico Esdras Muniz
Solução muito boa.

Em 27 de mai de 2017 00:37, "Gabriel Tostes"  escreveu:

> Tira ln, esse produto vai ser:
> Sum{n>=1} ln(n+1)/(2^n) = M
>
> Bora escrever M de outro jeito:
>
> M= ln(2) + [ln(3)-ln(2)]/2 + [ln(4)-ln(3)]/2^2 + ...
>
> M= Sum{n>=1} (ln(n+1)-ln(n))/2^(n-1)
>
> Como ln(n+1)-ln(n)=ln(1+1/n)<1/n
>
> M=2} 1/n.2^(n-1) = L + ln(2)
>
> Para achar L considere:
> 1/(1-x)= 1+x^2+x^3+...
>
> Integrando essa expressao temos que -(1/x).ln(1-x)= 1+x/2+x^2/3+...
> Substituindo x=1/2 achamos que L=2ln(2)-1
> E entao
> M< 3ln(2)-1 < ln(3)
>
>  E o produto pedido inicialmente eh menor que 3
>
>
>
>
>
>
>
>
> Sent from my iPad
> > On May 26, 2017, at 9:47 PM, Douglas Oliveira de Lima <
> profdouglaso.del...@gmail.com> wrote:
> >
> > Como posso fazer essa daqui:
> >
> > [2^(1/2)].[3^(1/4)].[4^(1/8)].[5^(1/16)]...<3
> >
> > Grande abraço a todos
> >
> > DouglasOliveira
> >
> > --
> > Esta mensagem foi verificada pelo sistema de antivírus e
> > acredita-se estar livre de perigo.
>
> --
> Esta mensagem foi verificada pelo sistema de antivírus e
>  acredita-se estar livre de perigo.
>
>
> =
> Instruções para entrar na lista, sair da lista e usar a lista em
> http://www.mat.puc-rio.br/~obmlistas/obm-l.html
> =
>

-- 
Esta mensagem foi verificada pelo sistema de antiv�rus e
 acredita-se estar livre de perigo.



Re: [obm-l] boatos sobre elon lages lima

2017-05-23 Por tôpico Esdras Muniz
Morreu sim, já faz alguns dias. :(

Em 23 de maio de 2017 21:00, Israel Meireles Chrisostomo <
israelmchrisost...@gmail.com> escreveu:

> É verdade que o Elon morreu?Fiquei chocado com essa notícia, o pessoal
> aqui poderia confirmar a veracidade dessa notícia?
>
> --
> Esta mensagem foi verificada pelo sistema de antivírus e
> acredita-se estar livre de perigo.




-- 
Esdras Muniz Mota
Mestrando em Matemática
Universidade Federal do Ceará

-- 
Esta mensagem foi verificada pelo sistema de antiv�rus e
 acredita-se estar livre de perigo.



Re: [obm-l] desigualdade

2017-05-07 Por tôpico Esdras Muniz
Se vc faz S(x,y,z)=x/(x+y) + y/ (y+z) + z/(z+x). S>x/(x+y+z) + y/ (x+y+z) +
z/(z+y+x)=1.

Por outro lado, se vc toma x=1; n=n e z= 1/n, fica:
S(n)=1/(n+1)+n/(n+1)+(1/n)/(1+1/n) e se vc faz n tender para o infinito,
S(n) tende para 1.

Em 7 de maio de 2017 23:58, Anderson Torres <torres.anderson...@gmail.com>
escreveu:

> x/(x+y) + y/ (y+z) + z/(z+x)
>
> 1/(1+y/x) + 1/ (1+z/y) + 1/(1+x/z)
>
> 1/(1+A) + 1/ (1+B) + 1/(1+C) com ABC=1
>
> talvez dê para prosseguir
>
>
>
> Em 2 de maio de 2017 14:21, Pedro José <petroc...@gmail.com> escreveu:
> > Se pelo menos dois números forem iguais é fácil mostrar que a soma dará
> 1,5
> > <= 2.
> >
> > Para x, y e z diferentes, vamos supor x < y <z, o que dará a maior soma
> é x
> > , y = x+1 e z= y+1 = x+2
> >
> > teremos: x/(2x+1) + y/(2y+1) + z/(2z-2)
> >
> > é fácil ver que x/(2x+1) < 0,5 e y/(2y+1) < 0,5 ==> x/(2x+1) + y/(2y+1 <1
> >
> > (2z-2)/z = 2 -2/z, como x,y,z >0 e x < y < z ==> z>=3 ==> (2z-2)/z > 1
> ==>
> > z/(2z-2) <1 ==> x/(2x+1) + y/(2y+1) + z/(2z-2) < 2
> >
> > x/(x+y) + y/ (y+z) + z/(z+x) <= x/(2x+1) + y/(2y+1) + z/(2z-2) ==>
> x/(x+y) +
> > y/ (y+z) + z/(z+x) < 2.
> >
> > O sinal de desigualdade deve estar invertido.
> >
> > Saudações,
> > PJMS
> >
> > Em 30 de abril de 2017 21:32, Gabriel Tostes <gtos...@icloud.com>
> escreveu:
> >>
> >> Nem vi a condição de q era positivo, de fato n vale.
> >>
> >> Sent from my iPad
> >>
> >> On Apr 30, 2017, at 3:53 PM, Douglas Oliveira de Lima
> >> <profdouglaso.del...@gmail.com> wrote:
> >>
> >> Observe quando x=2, y=3 e z=1 a desigualdade não funciona, logo não
> >> basta substituir x+y=a,Â
> >> x+z=b e y+z=c, na verdade acho que  funciona ao "contrário" x/(x+y) +
> >> y/ (y+z) + z/(z+x) <= 2.
> >> A não ser que seja outra questão como por exemplo:
> >> (x+y)/z +(x+z)/y +(y+z)/x >=6 o que daria certo.
> >>
> >> Grande abraço
> >>
> >> Douglas Oliveira.
> >>
> >> Em 30 de abril de 2017 10:46, marcone augusto araújo borges
> >> <marconeborge...@hotmail.com> escreveu:
> >>>
> >>> Se x, y, z são números positivos, prove que x/(x+y) + y/ (y+z) +
> >>> z/(z+x) > = 2
> >>>
> >>>
> >>> --
> >>> Esta mensagem foi verificada pelo sistema de antivírus e
> >>> acredita-se estar livre de perigo.
> >>
> >>
> >>
> >> --
> >> Esta mensagem foi verificada pelo sistema de antivírus e
> >> acredita-se estar livre de perigo.
> >>
> >>
> >> --
> >> Esta mensagem foi verificada pelo sistema de antivírus e
> >> acredita-se estar livre de perigo.
> >
> >
> >
> > --
> > Esta mensagem foi verificada pelo sistema de antivírus e
> > acredita-se estar livre de perigo.
>
> --
> Esta mensagem foi verificada pelo sistema de antivírus e
>  acredita-se estar livre de perigo.
>
>
> =
> Instru�ões para entrar na lista, sair da lista e usar a lista em
> http://www.mat.puc-rio.br/~obmlistas/obm-l.html
> =
>



-- 
Esdras Muniz Mota
Mestrando em Matemática
Universidade Federal do Ceará

-- 
Esta mensagem foi verificada pelo sistema de antiv�rus e
 acredita-se estar livre de perigo.



[obm-l] Re: [obm-l] Qual a maior potência?

2017-01-16 Por tôpico Esdras Muniz
4^53 = 2^106 > 2^105 = (2^7)^15 = (128)^15 > 125^15 = 5^45 > 5^44.

Em 16 de janeiro de 2017 13:14, Douglas Oliveira de Lima <
profdouglaso.del...@gmail.com> escreveu:

> Olá amigos , gostaria de uma ajuda pra um raciocínio diferente, por log eu
> já fiz.
>
> Qual a maior potência? 4^53 ou 5^44.
>
> --
> Esta mensagem foi verificada pelo sistema de antivírus e
> acredita-se estar livre de perigo.




-- 
Esdras Muniz Mota
Mestrando em Matemática
Universidade Federal do Ceará

-- 
Esta mensagem foi verificada pelo sistema de antiv�rus e
 acredita-se estar livre de perigo.



[obm-l] Re: [obm-l] Re: [obm-l] Re: [obm-l] Re: [obm-l] Re: [obm-l] Re: [obm-l] Re: [obm-l] Re: [obm-l] Polinômio irredutível em Z

2016-11-24 Por tôpico Esdras Muniz
6h149740p847418
>>>>>>>>>
>>>>>>>>>
>>>>>>> --
>>>>>>> Esta mensagem foi verificada pelo sistema de antiv?rus e
>>>>>>>  acredita-se estar livre de perigo.
>>>>>>>
>>>>>>>
>>>>>>> 
>>>>>>> =
>>>>>>> Instru?es para entrar na lista, sair da lista e usar a lista em
>>>>>>> http://www.mat.puc-rio.br/~obmlistas/obm-l.html
>>>>>>> 
>>>>>>> =
>>>>>>>
>>>>>>>
>>>>>>>
>>>>>>
>>>>>>
>>>>>> 
>>>>>> This message was sent using IMP, the Internet Messaging Program.
>>>>>>
>>>>>>
>>>>>>
>>>>>> --
>>>>>> Esta mensagem foi verificada pelo sistema de antivírus e
>>>>>> acredita-se estar livre de perigo.
>>>>>>
>>>>>>
>>>>>> 
>>>>>> =
>>>>>> Instruções para entrar na lista, sair da lista e usar a lista em
>>>>>> http://www.mat.puc-rio.br/~obmlistas/obm-l.html
>>>>>> 
>>>>>> =
>>>>>>
>>>>>
>>>>>
>>>>> --
>>>>> Esta mensagem foi verificada pelo sistema de antivírus e
>>>>> acredita-se estar livre de perigo.
>>>>
>>>>
>>>> --
>>>> Esta mensagem foi verificada pelo sistema de antivírus e
>>>> acredita-se estar livre de perigo.
>>>>
>>>
>>>
>>> --
>>> Esta mensagem foi verificada pelo sistema de antivírus e
>>> acredita-se estar livre de perigo.
>>>
>>
>>
>> --
>> Esta mensagem foi verificada pelo sistema de antivírus e
>> acredita-se estar livre de perigo.
>>
>
>
> --
> Esta mensagem foi verificada pelo sistema de antivírus e
> acredita-se estar livre de perigo.
>



-- 
Esdras Muniz Mota
Mestrando em Matemática
Universidade Federal do Ceará

-- 
Esta mensagem foi verificada pelo sistema de antiv�rus e
 acredita-se estar livre de perigo.



Re: [obm-l] Problema de geometria.

2016-11-02 Por tôpico Esdras Muniz
O que são essas "flechas"?

Em 2 de novembro de 2016 17:57, Douglas Oliveira de Lima <
profdouglaso.del...@gmail.com> escreveu:

> Olá amigos , preciso de uma ajuda na seguinte questão, na verdade a
> resolução porque já tentei muita coisa, já aprendi muita coisa com ela, mas
> mesmo assim não a resolvi.
>
> As três flechas dos três lados (cordas) de um triângulo ABC inscrito em
> uma circunferência de raio R
> valem 1, 2 e 3 calcular a área do triângulo.
>
>
>
> Qualquer ajuda será bem vinda. Obrigado.
>
> Att . Douglas Oliveira
>
> --
> Esta mensagem foi verificada pelo sistema de antivírus e
> acredita-se estar livre de perigo.




-- 
Esdras Muniz Mota
Mestrando em Matemática
Universidade Federal do Ceará

-- 
Esta mensagem foi verificada pelo sistema de antiv�rus e
 acredita-se estar livre de perigo.



RE: [obm-l] Divisibilidade Simultânea

2016-10-17 Por tôpico Esdras Muniz
Sim, m = n =1.

-Mensagem Original-
De: "Richard Vilhena" 
Enviada em: ‎17/‎10/‎2016 20:41
Para: "obm-l@mat.puc-rio.br" 
Assunto: [obm-l] Divisibilidade Simultânea

Gostaria que uma ajuda. Obrigado!


É possível encontrar inteiros m > 0, n > 0, tal que (n + 1)|(m2 + 1) e 
simultaneamente (m + 1)|(n2 + 1) ?


-- 
Esta mensagem foi verificada pelo sistema de antiv�rus e 
acredita-se estar livre de perigo. 
-- 
Esta mensagem foi verificada pelo sistema de antiv�rus e
 acredita-se estar livre de perigo.



[obm-l] Re: [obm-l] Re: [obm-l] Re: [obm-l] Recorrência

2016-10-16 Por tôpico Esdras Muniz
Muito obrigado.

Em 16 de outubro de 2016 20:49, Rodrigo Renji <rodrigo.uff.m...@gmail.com>
escreveu:

> Olá pessoal : )
> Estou escrevendo um material, se quiserem dar uma olhada, os links deixo
> abaixo
>
> ►(9.14) equações de diferenças ( recorrências lineares) I
> https://dl.dropboxusercontent.com/u/21174119/compartilhar/
> equacoesdiferencas.pdf
> ►(9.15) equações de diferenças ( recorrências lineares) II
> https://dl.dropboxusercontent.com/u/21174119/compartilhar/
> equacoesdediferenas2.pdf
>
> Em 16 de outubro de 2016 20:51, Jeferson Almir <jefersonram...@gmail.com>
> escreveu:
>
>> Principles and Techniques in Combinatorics
>> ( Chen chuan-chong ) acredito ser intermediário pra Phoda
>> Aí desses pesados existe o Introduction to Combinatorics e o
>> Problems in Combinatorics and Graph Theory ambos do renomado IOAN TOMESCU
>>
>> Em domingo, 16 de outubro de 2016, Esdras Muniz <
>> esdrasmunizm...@gmail.com> escreveu:
>>
>>> Olá amigos, gostaria que me passassem eferências de livros ou artigos
>>> que falem sobre recorrência. Dês de já obrigado.
>>>
>>> --
>>> Esdras Muniz Mota
>>> Mestrando em Matemática
>>> Universidade Federal do Ceará
>>>
>>>
>>>
>>> --
>>> Esta mensagem foi verificada pelo sistema de antivírus e
>>> acredita-se estar livre de perigo.
>>
>>
>> --
>> Esta mensagem foi verificada pelo sistema de antivírus e
>> acredita-se estar livre de perigo.
>>
>
>
> --
> Esta mensagem foi verificada pelo sistema de antivírus e
> acredita-se estar livre de perigo.
>



-- 
Esdras Muniz Mota
Mestrando em Matemática
Universidade Federal do Ceará

-- 
Esta mensagem foi verificada pelo sistema de antiv�rus e
 acredita-se estar livre de perigo.



[obm-l] Recorrência

2016-10-16 Por tôpico Esdras Muniz
Olá amigos, gostaria que me passassem eferências de livros ou artigos que
falem sobre recorrência. Dês de já obrigado.

-- 
Esdras Muniz Mota
Mestrando em Matemática
Universidade Federal do Ceará

-- 
Esta mensagem foi verificada pelo sistema de antiv�rus e
 acredita-se estar livre de perigo.



[obm-l] Re: [obm-l] Teoria dos Números

2016-09-26 Por tôpico Esdras Muniz
Vou dar só uma dica: 3|(10^k)+2 para todo K natural.

Em 26 de setembro de 2016 16:37, Ricardo Leão <leaoricardo...@gmail.com>
escreveu:

> Seja n um inteiro não negativo. Prove que o número formado colocando 2^n e
> 2^(n+1) lado a lado em qualquer ordem é um múltiplo de 3.
>
> Eu tentei resolver usando congruência, mas eu travei nessa questão.
>
> Por favor, algum colega poderia fazer a demonstração?
>
> --
> Esta mensagem foi verificada pelo sistema de antivírus e
> acredita-se estar livre de perigo.




-- 
Esdras Muniz Mota
Mestrando em Matemática
Universidade Federal do Ceará

-- 
Esta mensagem foi verificada pelo sistema de antiv�rus e
 acredita-se estar livre de perigo.



RE: [obm-l] Ajuda em Aritmética

2016-09-26 Por tôpico Esdras Muniz
Se p é um primo diferente de 5, os restos dos outros 2 por 5 são os mesmos que 
os de p^2-1 e p^2+1 respectivamente. Se os 3 números são primos, nenhum deles é 
múltiplo de 5. Daí o produto (p^2-1)(p^2+1) não pode ser múltiplo de 5. Mas 
esse produto é p^4-1. Mas o pequeno teorema de Fermat garante que 5 divide 
p^4-1 se p for diferente de 4. Aí o problema acaba.

Se vc não quiser usar o pequeno teorema de Fermat, é só verificar que para r=1, 
2, 3 e 4, onde r é o resto de p por 5, ou 4p^2-1 ou 6p^2-1 é múltiplo de 5. 

Acho a primeira solução melhor pq mostra de onde o autor tirou a idéia de fazer 
a questão.

-Mensagem Original-
De: "Marcelo de Moura Costa" 
Enviada em: ‎26/‎09/‎2016 06:19
Para: "obm-l@mat.puc-rio.br" 
Assunto: [obm-l] Ajuda em Aritmética

Bom dia a todos, um anulo me apresentou esse problema e confesso que pela dica 
não consegui interpretá-lo corretamente e fiquei muito curioso como o mesmo, 
será que alguém poderia me ajudar?

O problema é:

Mostre que somente para p=5, os números p, 4p^2+1 e 6p^2+1 serão primos. (Dica: 
analise os restos da divisão de p por 5) 


Agradeço a atenção.



-- 
Esta mensagem foi verificada pelo sistema de antiv�rus e 
acredita-se estar livre de perigo. 
-- 
Esta mensagem foi verificada pelo sistema de antiv�rus e
 acredita-se estar livre de perigo.



[obm-l] Re: [obm-l] Re: Demosntração desigualdade

2016-09-08 Por tôpico Esdras Muniz
Dei uma olhada rapida, acho que as desigualdades 1, 2 e 3 nem sempre valem,
pois a concavidade da função tangente depende do intervalo em que o angulo
está. Mas o principal motivo da sua prova estar errada é vc achar que o k
vai poder "alcançar" o n, isso não pode acontecer pois vc está fazendo um
limite com o n indo para o infinito enquanto o k é fixo, pode ser muito
grande, mas é fixo.

Em 7 de setembro de 2016 13:24, Israel Meireles Chrisostomo <
israelmchrisost...@gmail.com> escreveu:

> Por favor alguém que entendeu o que fiz pode me ajudar a entender o que eu
> fiz de errado?
>
> Em 7 de setembro de 2016 12:37, Israel Meireles Chrisostomo <
> israelmchrisost...@gmail.com> escreveu:
>
>> Alguém pode me esclarecer o erro cometido na minha demonstração para esse
>> problema aqui:
>>
>> http://math.stackexchange.com/questions/1917400/inequality-o
>> n-six-variables
>>
>> O cara deu um contra exemplo que a desigualdade é falsa, mas não vejo
>> nenhum errro na minha demonstração, alguém poderia me dizer qual é o erro?
>>
>
>
> --
> Esta mensagem foi verificada pelo sistema de antivírus e
> acredita-se estar livre de perigo.
>



-- 
Esdras Muniz Mota
Mestrando em Matemática
Universidade Federal do Ceará

-- 
Esta mensagem foi verificada pelo sistema de antiv�rus e
 acredita-se estar livre de perigo.



[obm-l] Re: [obm-l] Re: [obm-l] Re: [obm-l] nome de um quadrilátero

2016-08-10 Por tôpico Esdras Muniz
Agora que vi na wikipédia acredito que o nome seja mesmo deltoide ou pipa
(prefiro o segundo), mas quando ainda estava no ensino médio um professor
me falou que o nome disso era rombo e eu acreditei até hoje.

Em 10 de agosto de 2016 19:02, Bruno Visnadi <brunovisnadida...@gmail.com>
escreveu:

> De acordo com o próprio Wikipédia, o nome é 'Deltoide' ou 'Pipa'.
>
> Em 10 de agosto de 2016 18:43, Luís Lopes <qed_te...@hotmail.com>
> escreveu:
>
>> Sauda,c~oes, oi Esdras,
>>
>> Obrigado. Difícil imaginar isso pois rhombus
>>
>> em inglês parece ser losango.
>>
>>
>> https://pt.wikipedia.org/wiki/Losango
>>
>>
>> Não me lembro de ter visto esse nome rombo.
>>
>>
>> Os livros didáticos usam esse nome para kite ?
>>
>>
>> Luís
>>
>> --
>> *De:* owner-ob...@mat.puc-rio.br <owner-ob...@mat.puc-rio.br> em nome de
>> Esdras Muniz <esdrasmunizm...@gmail.com>
>> *Enviado:* quarta-feira, 10 de agosto de 2016 21:13:26
>> *Para:* obm-l@mat.puc-rio.br
>> *Assunto:* [obm-l] Re: [obm-l] nome de um quadrilátero
>>
>> Rombo.
>>
>> Em 10 de agosto de 2016 17:43, Luís Lopes <qed_te...@hotmail.com>
>> escreveu:
>>
>>> Sauda,c~oes,
>>>
>>>
>>> Qual o nome em português para o
>>>
>>> quadrilátero chamado de kite em inglês ?
>>>
>>>
>>>
>>> https://en.wikipedia.org/wiki/Kite_(geometry)
>>>
>>>
>>> Abs,
>>>
>>> Luís
>>>
>>>
>>>
>>> --
>>> Esta mensagem foi verificada pelo sistema de antivírus e
>>> acredita-se estar livre de perigo.
>>>
>>
>>
>>
>> --
>> Esdras Muniz Mota
>> Mestrando em Matemática
>> Universidade Federal do Ceará
>>
>>
>>
>> --
>> Esta mensagem foi verificada pelo sistema de antiv�rus e
>> acredita-se estar livre de perigo.
>>
>> --
>> Esta mensagem foi verificada pelo sistema de antivírus e
>> acredita-se estar livre de perigo.
>>
>
>
> --
> Esta mensagem foi verificada pelo sistema de antivírus e
> acredita-se estar livre de perigo.
>



-- 
Esdras Muniz Mota
Mestrando em Matemática
Universidade Federal do Ceará

-- 
Esta mensagem foi verificada pelo sistema de antiv�rus e
 acredita-se estar livre de perigo.



[obm-l] Re: [obm-l] nome de um quadrilátero

2016-08-10 Por tôpico Esdras Muniz
Rombo.

Em 10 de agosto de 2016 17:43, Luís Lopes <qed_te...@hotmail.com> escreveu:

> Sauda,c~oes,
>
>
> Qual o nome em português para o
>
> quadrilátero chamado de kite em inglês ?
>
>
>
> https://en.wikipedia.org/wiki/Kite_(geometry)
>
>
> Abs,
>
> Luís
>
>
>
> --
> Esta mensagem foi verificada pelo sistema de antivírus e
> acredita-se estar livre de perigo.
>



-- 
Esdras Muniz Mota
Mestrando em Matemática
Universidade Federal do Ceará

-- 
Esta mensagem foi verificada pelo sistema de antiv�rus e
 acredita-se estar livre de perigo.



RE: [obm-l] Fatorial e números primos

2016-04-01 Por tôpico Esdras Muniz
https://pt.m.wikipedia.org/wiki/Fatorial

Veja "Fatoração prima de fatoriais".

-Mensagem Original-
De: "Pedro Chaves" 
Enviada em: ‎01/‎04/‎2016 18:22
Para: "obm-l@mat.puc-rio.br" 
Assunto: [obm-l] Fatorial e números primos

Caros Colegas,

Proponho o teorema abaixo.

Teorema:

---  Na decomposição em fatores primos positivos do inteiro n >3,  o fator 2 
aparece mais vezes do que qualquer outro fator.  ---

Agradeço-lhes a atenção.

Pedro Chaves
---






-- 
Esta mensagem foi verificada pelo sistema de antivírus e 
acredita-se estar livre de perigo. 
-- 
Esta mensagem foi verificada pelo sistema de antiv�rus e
 acredita-se estar livre de perigo.



[obm-l] Re: [obm-l] Dúvida função

2016-01-30 Por tôpico Esdras Muniz
Teorema: Se f: R ---> Y é contínua e X é compacto, então f admite um máximo
e um mínimo em X.

Em 28 de janeiro de 2016 23:16, Israel Meireles Chrisostomo <
israelmchrisost...@gmail.com> escreveu:

> Olá pessoal eu gostaria de provar que uma função admite máximo sem
> calcular o máximo da função, isto é possível?
> Por exemplo, seja f(a,b,c) uma função, eu quero provar que a,b,c admite
> máximo sem calcular seu máximo, lembrando f(a,b,c) é uma função de 3
> variáveis, alguém por favor poderia me ajudar?
>



-- 
Esdras Muniz Mota
Mestrando em Matemática
Universidade Federal do Ceará


Re: [obm-l] Ajuda numa desigualdade.

2016-01-28 Por tôpico Esdras Muniz
L = ((1+1/(n+1))^(n+1))/(1+1/n)^n = ((1 - 1/(n+1)²)^n)((n+2)/(n+1))

Use que (1 - x)^n > 1 - nx, Para x \in (0, 1)

L > (1 - n/(n+1)²)((n+2)/(n+1)) = ((n²+n+1)/(n²+2n+1))((n+2)/(n+1))
= (n³+3n²+3n+2)/(n³+3n²+3n+1) > 1.



Esse último termo é maior que 1.

Em 28 de janeiro de 2016 09:41, Douglas Oliveira de Lima <
profdouglaso.del...@gmail.com> escreveu:

> Opa Marcelo, muito obrigado mesmo, eu estou procurando uma solução
> daquelas tipo
> desigualdades, onde efetuamos uma estratégia para chegar no resultado,
> tipo uma daquelas que tu encontra no livro de combinatória do MOrgado(o
> problema das apostas).
> Mas valeu, se conseguir uma dessas me manda novamente por favor.
> Abraço
> Douglas Oliveira
>
> Em 28 de janeiro de 2016 01:26, Marcelo Salhab Brogliato <
> msbro...@gmail.com> escreveu:
>
>> Oi, Douglas, tudo bem?
>>
>> Se provarmos que f(x) = (1 + 1/x)^x é estritamente crescente, então está
>> provada sua desigualdade.
>>
>> Uma maneira é fazer isso usando cálculo. Seja g(x) = ln(f(x)) = x ln(1 +
>> 1/x). Assim, se provarmos que g(x) é estritamente crescente, então f(x)
>> também será (exercício: prove essa afirmação).
>>
>> g'(x) = ln(1 + 1/x) + x * (-1/x^2) / (1 + 1/x)  = ln(1 + 1/x) - (1/x) /
>> (1 + 1/x) = ln(1 + 1/x) - 1/(1+x)
>>
>> Temos que mostrar que g'(x) > 0 para todo x.
>>
>> Sabemos que ln(x) < x - 1, para x != 1. Aplicando essa desigualdade em
>> 1/x, temos: ln(1/x) < 1/x - 1 => ln(x) > 1 - 1/x, para x != 1.
>>
>> Aplicando a desigualdade acima em 1+1/x, temos: ln(1+1/x) > 1 - 1/(1 +
>> 1/x) = (1/x) / (1 + 1/x) = 1/(1+x). Logo: ln(1+1/x) > 1/(1+x) => g'(x) > 0
>> para todo x (já que 1+1/x > 1).
>>
>> Abraços,
>> Salhab
>>
>> 2016-01-28 0:34 GMT-02:00 Douglas Oliveira de Lima <
>> profdouglaso.del...@gmail.com>:
>>
>>> Olá caros amigos, gostaria de uma ajuda na seguinte desigualdade
>>> (1+1/n)^n<(1+1/n+1)^(n+1), para n natural.
>>>
>>> Agradeço desde já.
>>>
>>>
>>
>


-- 
Esdras Muniz Mota
Mestrando em Matemática
Universidade Federal do Ceará


[obm-l] Re: [obm-l] Re: [obm-l] duas séries e um resultado

2016-01-11 Por tôpico Esdras Muniz
vc quer calcular limite quando n vai pro infinito de:

 \frac{ \sum_{k=0}^n \frac{1}{\sqrt{2k+1}} }{  \sum_{k=1}^n
\frac{1}{\sqrt{2k}} } + 1 =
\frac{ \sum_{k=1}^n \frac{1}{\sqrt{k}} }{  \sum_{k=1}^n \frac{1}{\sqrt{2k}}
} =
\sqrt{2}  \frac{ \sum_{k=1}^2n \frac{1}{\sqrt{k}} }{  \sum_{k=1}^n
\frac{1}{\sqrt{k}} } =
\sqrt{2}  [1 + \frac{ \sum_{k=n}^2n \frac{1}{\sqrt{k + n}} }{  \sum_{k=1}^n
\frac{1}{\sqrt{k}} } ] =
\sqrt{2} + \sqrt{2}  \frac{ \sum_{k=n}^2n \frac{1}{\sqrt{k + n}} }{
\sum_{k=1}^n \frac{1}{\sqrt{k}} }

Mas \frac{ \sum_{k=n}^2n \frac{1}{\sqrt{k + n}} }{  \sum_{k=1}^n
\frac{1}{\sqrt{k}} } vai pra zero com n, para ver basta usar que a média
dos quadrados é maior ou igual a média aritmética, assim:

\sum_{k=n}^2n \frac{1}{\sqrt{k}} \leq \frac{1}{n}\sqrt{\sum_{k=n}^2n
\frac{1}{k + n}} <

\frac{1}{n}\sqrt{\sum_{k=n}^2n \frac{1}{n}} = \frac{1}{n}.

Enquanto \sum_{k=1}^n \frac{1}{\sqrt{k}} > 1.


RE: [obm-l] Questão interessante

2015-11-03 Por tôpico Esdras Muniz
Se não há um dígito que aparece 3 vezes, então cada digito 0, 1, ..., 9 aparece 
duas vezes. Então a soma dos dígitos de p^n é 90, então 9|p^n.

-Mensagem Original-
De: "marcone augusto araújo borges" 
Enviada em: ‎03/‎11/‎2015 07:32
Para: "obm-l@mat.puc-rio.br" 
Assunto: [obm-l] Questão interessante

Seja p um número primo, p > 3.Sabe-se que para um certo inteiro positivo n
o número p^n possui 20 dígitos, quando escrito na base 10.Prove que dentre
esses dígitos existem pelo menos três iguais.


Eu tenho a solução.Estou compartilhando.

-- 
Esta mensagem foi verificada pelo sistema de antivírus e 
acredita-se estar livre de perigo. 
-- 
Esta mensagem foi verificada pelo sistema de antiv�rus e
 acredita-se estar livre de perigo.



RE: [obm-l] Matriz nxn

2015-11-03 Por tôpico Esdras Muniz
Dá (x-a)^{n-1}(x+(n-1)a). Eu fiz usando que esse determinante é um polinômio de 
grau n em x e coeficientes dependendo de a: "P^n(x,a)" (notação para o 
polinômio, de grau n,  do determinante desejado, em x e a). Daí temos que p(cx, 
ca)= c^nP(x, a). E, usando Chió, conseguimos: 
P^n(x,a)={(x-a)/x}^{n-1}P^{n-1}(x+a, a). O que implica que 
(x-a)^{n-1}|P^n(x,a). Agora, usando raízes (n-1)-esimas da unidade descobrimos 
que a outra raiz do polinômio é -(n-1)x.

Mas acho que conhecendo o resultado, deve ser mais fácil provar por indução...

-Mensagem Original-
De: "Anderson Torres" 
Enviada em: ‎03/‎11/‎2015 22:49
Para: "obm-l@mat.puc-rio.br" 
Assunto: Re: [obm-l] Matriz nxn

Você quer dizer algo assim, por exemplo?

X A A A A
A X A A A
A A X A A
A A A X A
A A A A X


Em 3 de novembro de 2015 23:42, Anderson Torres
 escreveu:
> Dê um exemplo. Não entendi nada.
>
> Em 3 de novembro de 2015 22:26, Eduardo Henrique
>  escreveu:
>> Pessoas, me deparei com a seguinte questão:
>>
>> Seja M uma matriz nxn com x na diagonal principal, e a>0 nas demais
>> posições. Calcule det(M).
>>
>> Alguém poderia me indicar um caminho para seguir? Eu não consegui avançar
>> nada nessa questão :(
>>
>> Att.
>>
>> Eduardo
>>
>> --
>> Esta mensagem foi verificada pelo sistema de antivírus e
>> acredita-se estar livre de perigo.

-- 
Esta mensagem foi verificada pelo sistema de antiv�rus e
 acredita-se estar livre de perigo.


=
Instru��es para entrar na lista, sair da lista e usar a lista em
http://www.mat.puc-rio.br/~obmlistas/obm-l.html
=

-- 
Esta mensagem foi verificada pelo sistema de antiv�rus e
 acredita-se estar livre de perigo.



RE: [obm-l] Aritmética

2015-10-24 Por tôpico Esdras Muniz
Suponha um impar i>1, tal que i|mdc(a,b). Daí p=x^i+y^i=(x+y)() e tem-se um 
absurdo.

-Mensagem Original-
De: "marcone augusto araújo borges" 
Enviada em: ‎24/‎10/‎2015 23:58
Para: "obm-l@mat.puc-rio.br" 
Assunto: [obm-l] Aritmética

Alguém poderia resolver?


Sejam a, b, n, m inteiros positivos e suponha que a^n + b^m seja
um número primo.Mostre que (n,m) = 1 ou (n,m) = 2^r, para algum
r inteiro positivo.
Desde já agradeço.





-- 
Esta mensagem foi verificada pelo sistema de antivírus e 
acredita-se estar livre de perigo. 
-- 
Esta mensagem foi verificada pelo sistema de antiv�rus e
 acredita-se estar livre de perigo.



RE: [obm-l] Problema 6 da OBM de 2002

2015-10-12 Por tôpico Esdras Muniz
Em qual EUREKA está a solução deste problema?


-Mensagem Original-
De: "Bernardo Freitas Paulo da Costa" 
Enviada em: ‎12/‎10/‎2015 12:29
Para: "Lista de E-mails da OBM" 
Assunto: Re: [obm-l] Problema 6 da OBM de 2002

2015-10-12 0:31 GMT-03:00 Gabriel Tostes :
> Mostre que não podemos formar mais que 4096 sequências binárias de tamanho 24 
> tal que quaisquer 2 diferem em ao menos 8 posições.
> Não consegui entender a resolução na Eureka. Alguém pode resolvê-lo?

Eu não sei se conheço alguma solução além da do Fábio (imagio que seja
esta a da Eureka). Mas acredito que pode ajudar a entender o problema
diminuindo os números, e tentando ser mais ambicioso: tente descobrir
a maior quantidade de sequências binárias de tamanho 4 tais que duas
quaisquer diferem em ao menos 2 posições. Vou tentar começar o
raciocínio: Suponha, sem perda de generalidade, que uma das sequências
é a . Então, você não pode ter nenhuma sequência com apenas um
"1", certo? Agora, pense em quantas sequências com dois "1" podem
haver. Ao todo, há 6, mas você não pode escolher todas elas, certo?

Para completar, ainda falta a idéia das "regiões de influência" (cada
sequência escolhida "domina" algumas sequências, as que estão mais
próximas dela do que de qualquer outra sequência). Para visualisar
isso, pense que, em vez de 4/2, o problema é sobre tam=5 / diferença
>= 3. Faça um "ponto" para cada sequência (dá 32, dá trabalho mas é
factível) e ligue as que diferem de apenas uma posição. Daí, comece
marcando uma (tipo a 0) e depois vá escolhendo como puder. Isso
deve deixar claro para você que cada sequência tem uma região de
influência de tamanho 1.

Abraços,
-- 
Bernardo Freitas Paulo da Costa

-- 
Esta mensagem foi verificada pelo sistema de antiv�rus e
 acredita-se estar livre de perigo.


=
Instru��es para entrar na lista, sair da lista e usar a lista em
http://www.mat.puc-rio.br/~obmlistas/obm-l.html
=

-- 
Esta mensagem foi verificada pelo sistema de antiv�rus e
 acredita-se estar livre de perigo.



RE: [obm-l] Desigualdade

2015-10-08 Por tôpico Esdras Muniz
Suponha spdg a>=d>=c. Daí, pela desigualdade do rearranjo, temos:
a(a^2/bc)+b(b^2/ac)+c(c^2/ab)>=(1/3)(a+b+c)(a^2/bc+b^2/ac+c^2/ab).
Daí vc usa MA>=MG pra mostrar que (a^2/bc+b^2/ac+c^2/ab)>=3. E acaba :)

-Mensagem Original-
De: "marcone augusto araújo borges" 
Enviada em: ‎08/‎10/‎2015 18:03
Para: "obm-l@mat.puc-rio.br" 
Assunto: [obm-l] Desigualdade

Sejam a, b e c números reais positivos.Mostre que
a^3/bc + b^3/ac + c^3/ab > = a + b + c

-- 
Esta mensagem foi verificada pelo sistema de antivírus e 
acredita-se estar livre de perigo. 
-- 
Esta mensagem foi verificada pelo sistema de antiv�rus e
 acredita-se estar livre de perigo.



[obm-l] Re: [obm-l] questão de treinamento olimpica.

2015-10-07 Por tôpico Esdras Muniz
Supondo por absurdo que isso ocorra, daí  temos que se a_i=11, então
b_i=11, do contrario, teríamos dois produtos de resto zero por 11. Então
vamos supor sem perda de generalidade que a_11=b_11=11.
daí, se x_i=a_i.b_i, supondo que {x_1,..., x_10} têm todos os restos
positivos possíveis por 11, então x_1.x_2...x_10 é congruente a (1.2...10)
(mod 11). (1.2...10)=(11-1)!.
Mas: x_1.x_2...x_10=(1.2...10)².
O teorema de Wilson garante que:
(11-1)! é congruente a -1 (mod 11). enquanto que (1.2...10)² é congruente a
1 (mod 11). O que é um absurdo.

Em 7 de outubro de 2015 08:59, Mauricio de Araujo <
mauricio.de.ara...@gmail.com> escreveu:

> Sejam a1, a2, ..., a11 e b1, b2, ..., b11 duas permutações dos inteiros 1,
> 2, ..., 11. Considere os números a1.b1, a2.b2, a3.b3, ..., a11.b11. Mostre
> que pelo menos dois destes números deixam o mesmo resto quando divididos
> por 11.
> Sugestão: Redução ao absurdo.
>
> --
> Abraços
>
> oɾnɐɹɐ ǝp oıɔıɹnɐɯ
>
>
> --
> Esta mensagem foi verificada pelo sistema de antivírus e
> acredita-se estar livre de perigo.




-- 
Esdras Muniz Mota
Mestrando em Matemática
Universidade Federal do Ceará

-- 
Esta mensagem foi verificada pelo sistema de antiv�rus e
 acredita-se estar livre de perigo.



  1   2   >